You are on page 1of 212

R

TABLE OF CONTENTS

LIST OF FIGURES
AUTHOR:
Figure 1-Satelite map of Kalang'oma village ..... Error! Bookmark not defined.
Figure 2- A small marsh near Nankhaka river .. Error! Bookmark not defined.
Figure 3- Sketch of village profile .................... Error! Bookmark not defined.
Figure 4- Gule Wamkulu ................................ Error! Bookmark not defined.
PAEDS ACROSS
Figure 5-Jehovah's witness church and the village cemeteryError! Bookmark not
defined.
YEARS
Figure 6- Earth road and ox-drawn cart .......... Error! Bookmark not defined.
Figure 7-Grass thatched house ........................ Error! Bookmark not defined.
Figure 8- Boys playing soccer and pool table... Error! Bookmark not defined.
SUBMITTED
Figure 9- Community TO MBBS 3 STUDENTS
members drawing water from a boreholeError! Bookmark
not defined.
Figure 10-Ablution facilities............................. Error! Bookmark not defined.
Figure 11-Inside the ablution facilities and improper waste disposal in the village
..................................................................... Error! Bookmark not defined.
Figure 12- Women preparing vegetables for sale and men TROY MADAH
doing piece workError!
Bookmark not defined. KAMPIRA
Figure 13- A man on his way to sale maize and a food standError! Bookmark not
defined.
MBBS 3

201850044596

0994679938
2022 END OF 3RD PAEDIATRICS ROTATION

What would be the initial management of Chisomo?


• Make sure the airway is patent, that there is adequate respiratory effort and
that there is adequate circulatory perfusion.
• Put child in decubitus position
• Check blood glucose, if hypoglycemia give dextrose D10% 5ml/kg IV
• If no hypoglycemia give diazepam 0.15-0.25mg/kg IV or 0.5mg/kg rectally
Which investigations would you do?
• RBS
• U&Es
• LFT
• LP
• MRDT
• Blood culture
• HTC
• MPs
• FBC
• CHEST X-RAY

1|Page ...TROY MADAH KAMPIRA...


What is the most likely diagnosis
• acute poststreptococcal glomerulonephritis

What is the mechanism of the oedema


• There is urinary loss of proteins which leads to hypoalbuminemia and this
reduces the plasma oncotic pressure? As a result, there is transudation of
fluid from the intravascular compartment leading to a decrease in
intravascular volume. A decrease in intravascular volume causes a fall of
the renal perfusion pressure and this activates the renin-angiotensinaldosterone
system which stimulates the tubular reabsorption of sodium
thereby causing oedema.

2|Page ...TROY MADAH KAMPIRA...


Explain how RESOMAL is different from ORS and why this is relevant in a
malnourished child in the ward
• ReSoMal has less sodium, more potassium and glucose than ORS. It is relevant
for the malnourished child because they have a lot of sodium and hypokalemia in
their bodies.

Explain the pathogenesis of diarrhea in malnutrition


• Malnutrition causes atrophy of the villi and epithelial cells of the bowel mucosa
hence the absorption of water and nutrients is reduced and this leads to diarrhea.
• Malnutrition also reduces immunity which predisposes to different infections
that may cause diarrhea.

Explain how HIV may cause malnutrition


HIV causes increased energy expenditure- there is increase in resting energy
expenditure (or the resting metabolic rate/RMR). Children require a higher
proportion of energy for growth and development. HIV causes malabsorption of
nutrients from the intestines- food may be poorly absorbed in patients with HIV
and AIDS. Intestinal malabsorption and nutrient loss are common. The virus has
3|Page ...TROY MADAH KAMPIRA...
been shown to damage the intestinal villi, and inflammation can damage gut
tissue and reduce absorption. These changes in the gut seem to affect the body’s
ability to utilize dietary fat and carbohydrates.

• People with HIV and AIDS often eat less — most commonly because of a loss
of appetite. In addition to underlying HIV, a number of opportunistic infections
contribute to this by causing nausea, malaise, and fever. Infections, such as
esophageal candidiasis, that cause a sore mouth or pain from eating, also decrease
food intake — and this may occur silently in children

• “HIV seems to induce a special metabolic effect involving a preferential loss of


protein over fat

4. Please describe 3 features of bruising that would be the concerning for non-
accidental trauma
i. They occur on more than one body surface
ii. They are in multiple stages of healing
iii. They are patterned or well demarcated

5. List Four metabolic derangements in tumor lysis

i. Hyperkalemia
ii. Hyperphosphatemia
iii. Hyperuricemia
iv. Hypocalcemia

4|Page ...TROY MADAH KAMPIRA...


6. List 4 possible symptoms for Wilms tumor
a. G.I symptoms
i. Palpable large mass in the abdomen
ii. Abdominal distention
iii. Constipation

b. Renal symptoms
i. Hematuria
ii. Urinary tract infections
iii. Pain in flanks

c. Cardio-vascular symptom
i. Hypertension

d. Respiratory symptom
i. Shortness of breath

e. General symptoms
i. Loss of appetite
ii. Fever
iii. Nausea and vomiting
Anorexia & general malaise

5|Page ...TROY MADAH KAMPIRA...


7. Four common infectious complications of atopic dermatitis associated
with dermatitis

i. Impetigo: secondary to staph. aureus & some from Strep. group A


ii. Eczema herpeticum: secondary to HSV
iii. Molluscum contagiosum: secondary to Poxvirus
iv. Dermatophyte infection: Tinea

8. Describe the pathophysiology of physiological jaundice


Neonatal physiological jaundice results from simultaneous occurrence of the
following two phenomena:
i. Bilirubin production is elevated because of the increased
breakdown of fetal erythrocytes. This is the result of the shortened
lifespan of fetal erythrocytes and the higher erythrocyte mass in
neonates
ii. Hepatic excretory capacity is low both because of low
concentrations of the binding protein ligandin (Glutathione-S-
Transferase) in the hepatocytes and because of low activity of
Uridine-Di-Phospho-Glucuronyl Transferase (UDPGT), the enzyme
responsible for binding bilirubin to glucuronic acid, thus making
bilirubin water soluble (conjugation).

6|Page ...TROY MADAH KAMPIRA...


2022 END OF 2ND PAEDIATRICS ROTATION

1. Please outline 4 risk factors for tuberculosis in children


i. Household or other close contact of a case of pulmonary TB most
especially those with smear-positive or culture-positive pulmonary TB
ii. Age less than five years
iii. HIV infection
iv. Severe malnutrition

2. Please describe 3 features of bruising that would be the concerning for


non-accidental trauma

3. List 6 pulmonary causes of Respiratory distress in new born babies

4. Atopic dermatitis, what else would you recommend for management of


this exacerbation besides bathing daily and non-soap cleanses

i. Apply topical corticosteroids such as hydrocortisone twice daily for 5 to


7 days which has anti-inflammatory properties
ii. Followed by two successful layers of cotton pajamas, gauze or tubular
bandages (first layer wetted with warm water; second layer dry)

5. Four common infectious complications of atopic dermatitis associated with


dermatitis

7|Page ...TROY MADAH KAMPIRA...


6. Please describe the mechanism by which ORS functions as treatment for
dehydration in patients with acute gastroenteritis

The mechanism of action of ORS involves the sodium-glucose transporter


which co-transports one sodium with one glucose. Water then follows by
diffusion because of the concentration gradient of sodium.

ORS has relatively low carbohydrate load and elevated sodium and potassium
levels that maximize hydration while minimizing osmotic loads that drive
diarrhea.

7. List 4 possible symptoms for Wilms tumor

8. List Four metabolic derangements in tumor lysis

9. Describe the pathophysiology of physiological jaundice

2022 END OF 1ST PAEDIATRICS ROTATION


(same as above 2022 end of 2nd Paeds rotation except question about jaundice)

8|Page ...TROY MADAH KAMPIRA...


2021 END OF 1ST PAEDIATRICS ROTATION SAQs
1. Concerning Refeeding syndrome
a. List 4 metabolic derangements of refeeding syndrome (4 marks)
i. Hypokalemia
ii. Hypophosphatemia
iii. Hypomagnesemia
iv. Hypernatremia
b. List clinical features of refeeding syndrome (6 marks)
i. Rhabdomyolysis
ii. Neutrophil dysfunction
iii. Cardio-respiratory failure
iv. Arrhythmias
v. Seizures
vi. Altered level of consciousness, or sudden death.

2. In a neonate with jaundice, what is the rationale of the following


components in exam? (2 marks each)
a. Wellness
i. Wellness is assessed to help in differentiating whether the jaundice
is physiological or pathological.
ii. It is also used to assess whether the newborn is preterm or term

b. Colour
Colour is assessed to trace the severity of the jaundice which is the
indication for phototherapy.
[Bilirubin check: Palms and sores (15-20mg/dl), Arms and lower limbs
{15-18mg/dl), Abdomen (12-14mg/dl), Chest (8-10mg/dl), Face (5-
6mg/dl)]

9|Page ...TROY MADAH KAMPIRA...


c. Neurological
To assess if the bilirubin has crossed blood-brain barrier and affected the
brain. For example, in Kernicterus which may lead to complication like
cerebral palsy

d. Umbilical
To check for infections i.e., to check if jaundice is accelerated with
infections

e. Physical bruises
To check for entry of infections through bruises
Bruises may also show congenital syphilis

3. Concerning child’s obesity


a. What are the impacts of the condition in: (2marks on each)
i. Respiratory
• Obese children may be asthmatic because fat tissue produces
inflammatory substances that might affect the lungs
• Excess fats reduce total respiratory system compliance,
increases pulmonary resistance, and reduces respiratory
muscle strength
• Obesity causes sleep apnea

ii. Metabolism
• There is impaired glucose intolerance
• Insulin resistance, Diabetes type 2

iii. Musculoskeletal
• Obesity leads to joint problems
• It causes muscular-skeletal discomfort
10 | P a g e ...TROY MADAH KAMPIRA...
iv. Psychosocial
• It leads to psychogenic stress; depression
• It leads to behavior problems
• It leads to bullying issues at school which lowers one’s self
esteem and self-reported quality of life

v. Cardio-vascular
• Obesity accelerates high blood pressure
• Obesity leads to accumulation of more cholesterol in the
body leading to high risk of CVD

4. 4 weeks old male presented with projectile non-bilious vomiting for 2 days
a. What is the likely diagnosis? (2marks)
i. Duodenal atresia

b. List 4 metabolic derangements of the condition above (4 marks)


i. Hypokalemia
ii. Hypochloremia
iii. Hypernatremia
iv. Metabolic alkalosis
c. How would you confirm the diagnosis? (1mark)
Abdominal Ultrasound

d. What is the management? (3 marks)


i. Supportive Management: Administer IV fluids, prepare the child
for surgery
ii. Definitive Management: Treated surgically through
duodenoduodenostomy; a new connection between the upper
duodenum and distal duodenum is created to bypass the
blockage
11 | P a g e ...TROY MADAH KAMPIRA...
2021 END OF 3RD PAEDIATRICS ROTATION SAQs

ANSWERS TO QUESTIONS ABOVE


1. Management of Hyper-cyanotic spells inn TOF
12 | P a g e ...TROY MADAH KAMPIRA...
a. Put the child in a squatting or knee-chest position – to increase systemic
vascular resistance and promote systemic venous return to the heart
b. Give oxygen - to decrease peripheral pulmonary vasoconstriction
c. Analgesia (e.g., morphine)- to decrease release of catecholamines
thereby decrease heart rate (increasing filling time) and relaxation of the
infundibular spasm
d. IV fluid bolus - to improve right ventricular preload

2. Dehydration states that require cautious replacement of IV fluids:


a. Severe dehydration
b. Some dehydration

3. Effective communication can affect the patient care in following ways:


a. Patient disclosure
b. Accurate diagnosis
c. Treatment adherence and outcome
d. Adaptation to illness
e. Bereavement

4. Diagnostic and Statistical manual of Mental Disorders 5th Edition (DSM-V)


criteria for Attention Deficit Hyperactivity Disorder (ADHD):
a. Must have symptoms and/or behaviors for at least 6 months in at least 2
settings (home, school, church)
b. These symptoms negatively impact academic, social, and/or
occupational functioning
c. Must have at least 6 symptoms
d. Symptoms present from prior to age 12
e. Symptoms are not better accounted for by a different psychiatric
disorder

13 | P a g e ...TROY MADAH KAMPIRA...


5. Differences between Caput succedaneum, Subgaleal hemorrhages and
Cephalohematoma
a. Caput succedaneum
i. It is diffuse subcutaneous fluid collection with poorly defined
margin often crossing the suture lines caused by pressure on the
presenting part of the head during delivery.
ii. It does not usually cause complications and resolves over the
first few days

b. Subgaleal hemorrhages
i. It occurs between the scalp and the periosteum and form a
large, fluctuant collection which crosses suture lines.
ii. They are rare but may cause life-threatening blood loss

c. Cephalohematoma
i. It is a sub-periosteal hemorrhage which occurs in 1-2% of
infants and may increase in size after birth.
ii. The hemorrhage is bound to periosteum, the swelling does
not cross the suture line in contrast to a caput succedaneum
iii. More common with instrumental delivery and may cause
jaundice

6. a. The immunity is compromised in Sickle cell disease because:


i. There is impaired leucocyte function
ii. There is humoral and cell-mediated immunity loss
iii. SCD is associated with auto-splenectomy which increases
susceptibility to infections

b. They are most associated with invasive bacterial infections (IBI) like
14 | P a g e ...TROY MADAH KAMPIRA...
i. Streptococcus pneumoniae
ii. Non-typhoidal Salmonella species
iii, Haemophilus influenzae type B (Hib)

7. Infectious complications of Atopic Dermatitis


i. Impetigo: secondary to staph. aureus & some from Strep. grp A
ii. Eczema herpeticum: secondary to HSV
iii. Molluscum contagiosum: secondary to Poxvirus
iv. Dermatophyte infection: Tinea

2021 - END OF 4TH PAEDIATRICS ROTATION SAQs


1. CEREBRAL PALSY CASE:
a. Define CP
It is a non-progressive disorder of motion and/or posture, secondary
to an insult in the developing brain

b. Patient with Weight for Height < -3 SD, with no edema: what is the
dx and how can u manage him

c. Anticipate 2 problems in this patient and where are you going to


refer them
i. Contractures and other deformities that need Physiotherapists
and occupational therapists
ii. Kyphoscoliosis and dislocation of bones that require
Orthopaedical surgeons
iii. Myopia, squint that require Ophthalmologists

15 | P a g e ...TROY MADAH KAMPIRA...


iv. Sensorineural deafness and problems in language
comprehension that require Speech therapists

d. What is the patient expected to be doing if he was 3 years old?

2. CHRONIC RENAL FAILURE:


a. List 4 complications of chronic renal failure and how are they caused
v. Hyperkalemia
vi. Fluid overload
vii. Anaemia
viii. Renal osteo-dystrophy

b. Describe how you can manage hyperkalemia


i. Rapidly decrease the risk of life-threatening arrhythmias

ii. Shift potassium intracellularly:


• Sodium bicarbonate administration (IV)
• Insulin and glucose (IV)
• β-agonist (salbutamol via nebulizer)

iii. Cardiac membrane stabilization


• IV calcium

iv. Remove potassium from the body


• Loop diuretic (IV or PO)
• Sodium polystyrene (PO or rectal)
• Dialysis

3. MICRONUTRIENTS:
a. Mention Two common micronutrient deficiencies in Malawi
i. Iron deficiency

16 | P a g e ...TROY MADAH KAMPIRA...


ii. Vitamin A deficiency
iii. Iodine deficiency

b. What are risk factors for the micronutrient deficiencies?


i. Inadequate breastfeeding
ii. Increased physiological demands for growth during pregnancy
and lactation
iii. High incidence of infectious diseases such as intestinal parasitic
infections
iv. Low intake due to poverty, lack of access to variety of foods
and lack of knowledge of optimal dietary practices

c. What interventions are being done in Malawi?


i. Iron tablets supplement in antenatal visit to pregnant
mothers
ii. Give Vitamin A every month up to 5 years to children
iii. Iodization of salt

4. ASTHMA CASE:
a. What is the Immediate management of asthma?
Grade the Asthma and the management will depend on the grade:

i. Intermittent
• Salbutamol as needed (short acting beta-agonist, SABA)

ii. Mild persistent


• SABA as needed
• Daily low-dose inhaled corticosteroid (ICS) =
beclomethasone

iii. Moderate persistent

17 | P a g e ...TROY MADAH KAMPIRA...


• SABA as needed
• Daily medium-dose ICS

Iv. Severe persistent


• SABA as needed
• Daily high-dose ICS or Daily medium-dose +leukotriene
receptor antagonist (LTRA)

b. How will you manage the child on discharge?

If the patient is no longer requiring oxygen and is able toto eat and
drink discharge him with the following managements
i. 4 hours in between salbutamol treatments
ii. Continue around the clock salbutamol for next 48 hours then
gradually space out
iii. 2nd dose of dexamethasone 36 hours after the first dose OR
iv. 3-5 days of prednisone (2 mg/kg first dose then 1 mg/kg BD

2020 END OF YEAR PAEDIATRICS MCQs

18 | P a g e ...TROY MADAH KAMPIRA...


ANSWERS: 1. C 2. E

19 | P a g e ...TROY MADAH KAMPIRA...


20 | P a g e ...TROY MADAH KAMPIRA...
ANSWERS: 3. H 4. G 5. F 6. A

ANSWERS: 13. F 14. H 15. E

21 | P a g e ...TROY MADAH KAMPIRA...


ANSWER: 19.E

22 | P a g e ...TROY MADAH KAMPIRA...


ANSWERS: 20. A 21. E

23 | P a g e ...TROY MADAH KAMPIRA...


ANSWERS: 22. C 23. D

24 | P a g e ...TROY MADAH KAMPIRA...


ANSWERS: 24. B 25. E 26. E

25 | P a g e ...TROY MADAH KAMPIRA...


ANSWER: 27. E

26 | P a g e ...TROY MADAH KAMPIRA...


27 | P a g e ...TROY MADAH KAMPIRA...
ANSWERS: 28. D 29. C

28 | P a g e ...TROY MADAH KAMPIRA...


ANSWER: 30. A

29 | P a g e ...TROY MADAH KAMPIRA...


30 | P a g e ...TROY MADAH KAMPIRA...
ANSWER: 31. A

31 | P a g e ...TROY MADAH KAMPIRA...


ANSWERS: 32. B 33. A

2020 END OF YEAR PAEDIATRICS SAQs


1.A 28-day old male infant presented in the under 5 clinics with history of
vomiting for 1 week. The baby was born at term with no significant antenatal
history. His birth weight was 3.3kgs, he was breast fed since birth, tolerated all his
feeds up until a week ago. The mother described the vomit as projectile occurring
20 minutes after each feed. She said that the vomits have always been milky
white never bilious/bloody. She also mentions that the baby has remained eager
to feed but his urine output and stool frequency has markedly decreased. On

32 | P a g e ...TROY MADAH KAMPIRA...


examination, he is thin, crying PR170bpm, moderately dehydrated, abdomen is
normal, iv access is obtained and initial blood tests are taken

a. Mention 3 electrolyte imbalances which are unlikely to be found


i. Hypernatremia
ii. Hypokalemia
iii. Hypochloremia
iv. Metabolic alkalosis

b. Mention one most appropriate step to confirm the diagnosis


i. Abdominal ultrasound scan

c. Immediate step management of this patient


i. IV bolus: Ringase Lactate

d. Differential diagnosis of this condition


i. Pyloric stenosis
ii. Duodenal atresia

2. (i) A 3-year-old boy brought to the clinic because of cough for the last 6
weeks. He was previously well, nonproductive cough day and night and
worsened by exertion.no body in the family is coughing.

a. 6 things you would ask in your history


i. Exposures
ii. Triggers- perfume, animals
iii. Interruption of daily activities
iv. Foreign body
v. Family history
vi. Exacerbations (days and nights)
vi. HIV status
vii. Weight loss
33 | P a g e ...TROY MADAH KAMPIRA...
viii. B symptoms
ix. Fever
x. TB contacts
v. On examination, scattered crepitation over both lung fields,
cervical lymphadenopathy and spleen 2cm below coastal
margin, mention 4 further investigations you will do
i. Chest X-ray
ii. Abdominal ultra-sound
iii. Tuberculin skin test
iv. Lymph node Biopsy
v. Gamma-interferon

(II) a previously health 24 months old child seen in the emergency department in
the morning hours for marked SOB, stridor, rhinorrhea, low grade fever and
brassy/harsh cough
a. What criteria should you use to determine initial management
i. A, B, C, D approach
• Access his Airway
• Access his Breathing
• Access his Circulation
• Don’t forget to give him glucose if indicated

b. What is the most likely cause of this clinical picture?


i. Upper airway obstruction
ii. Croup
iii. Diphtheria
iv. Pertussis

c. How should this child be treated?


i. Supportive management
• Put the child on oxygen
• Give him corticosteroids

34 | P a g e ...TROY MADAH KAMPIRA...


• Give him antihistamines

3.Tionge is 3 months old exclusively breast fed who is being treated for severe
pneumonia and today tested positive for HIV rapid antibody test.

a. Describe the possible meaning of this result and explain


The child is exposed
There is Presumptive Severe HIV Disease (PSHD)
b. What type of confirmatory test does the infant need and why?
DNA PCR
c. Describe three PMTCT strategies that should have protected this
infant
• Provider initiated early HIV testing for early diagnosis and ART
initiation
• Life-long ART for all HIV infected pregnant and breastfeeding
women
• Safe obstetric practices
i Provision of infant nevirapine prophylaxis
ii Exclusive breastfeeding in the first 6 months

d. The mother tested HIV in Paediatrics ward and, she was positive; in
her health passport you note that her 1st trimester test was negative.
what’s the significance of the information?
New infection with high-risk exposure for the baby

2019 END OF PAEDIATRICS ROTATION SAQs


1) List the four major developmental domains (4 marks)

i) Gross motor
ii) Fine motor and Vision
35 | P a g e ...TROY MADAH KAMPIRA...
iii) Hearing and Speech
iv) Social

a) For each of the four developmental domains give one example of


what you would expect a one-year-old child with normal
development to be able to do in that domain. (4 marks)

Developmental What you would expect a one-year-old to be


Domain able to do
Gross motor Walk, crawl, sit, pull to stand, cruise

Fine motor/ Pick up small objects with pincer grip, 2 small


vision objects in one hand, put small objects into cup,
turns pages of book, holds pencil, Looks at
pictures in a book
Hearing/ Understands bye-bye/no. One word with
Speech meaning, knows name, understands simple
instructions, few words, points at objects
Social Stranger anxiety, feeds from a cup, feeds self,
plays games

b) List four common causes of developmental delay in Malawi (2 marks)


❖ Perinatal asphyxia
❖ Meningitis
❖ Cerebral Malaria
❖ Down’s syndrome
❖ Cerebral palsy
❖ Chronic disease (HIV, TB)
❖ Malnutrition

2) a. State the characteristics of upper motor neuron (UMN) lesions


❖ ↑ in muscle tone
❖ ↑ tendon reflex
❖ Little or no atrophy
36 | P a g e ...TROY MADAH KAMPIRA...
❖ Extensor plantar response
❖ Clonus
❖ Spasticity

b. Characteristics of Lower motor neuron (LMN) lesions


❖ ↓ muscle tone
❖ Absent tendon reflexes
❖ Muscle wasting
❖ Fasciculations in affected muscle

3) a. Define cerebral palsy


A non-progressive disorder of motion and/or posture, secondary to
an insult in the developing brain

b. Common causes of Cerebral Palsy in Malawi


❖ Congenital brain abnormalities
❖ Birth asphyxia
❖ Neonatal hypoglycemia
❖ Kernicterus
❖ Cerebral malaria/ meningitis
❖ Early head injury

4) A 4-year-old girl is admitted because of acute gastroenteritis. The intern


recognizes that the child is severely dehydrated, and has a 6 cm spleen
a. Mention signs of severe dehydration
❖ Two or more of lethargy or comma
❖ Sunken eyes
❖ Unwilling or unable to drink
❖ Skin pinch goes back very slowly>2sec
❖ Capillary refill time >3seconds

b. State the etiologies of watery diarrhea


❖ Rotavirus
37 | P a g e ...TROY MADAH KAMPIRA...
❖ Vibrio cholerae
❖ Shigella
❖ Escherichia coli
❖ Salmonella typhi

c. What is the cause of watery diarrhea in a small community?


❖ Vibrio cholerae which may come due to lack of food hygiene, unsafe
water source
d. List three signs that will differentiate a kidney from a spleen on
abdominal palpation. (3 marks)

• Spleen is in the left hypochondrium while the kidney is in the loin


• Spleen has well defined medial border while spleen has a round upper end
• Spleen has a notch while kidney does not have a notch

Palpable spleen Left kidney

It is in the left hypochondrium In the left lumber region or loin

Moves with respirations towards RIF Moves downwards and forward

Notch is present No notch present

Get above the swelling possible Impossible

Insinuation of finger between the mass Possible


and left costal margin impossible

On percussion, dullness over the mass On percussion, colonic resonant over


which is continuous with the left lower the mass
chest

The mass is palpable The mass is palpable as well as


ballotable

38 | P a g e ...TROY MADAH KAMPIRA...


5. Respiratory distress-severe pneumonia. 18 months old child
(a) Define bronchial breathing
❖ Breath sounds of a harsh or blowing quality, heard on auscultation of
the chest made by air moving in the large bronchia and barely if at
all, modified by the intervening lung

❖ Hollow blowing sound heard due to increased density of matter in


the peripheral lung.

(b) Upper limit of respiratory rate (RR) of this child


❖ 40 breaths/min (for a normal child)

(c) The SO2 is low. The doctor examining the child knows that the oxygen
dissociation curve will be shifted to the right. List three factors that will shift
this curve to the right
❖ ↑ H+ (Bohr shift)
❖ ↑ temperature
❖ ↑ p(CO2)
❖ ↑ 2, 3-Biphosphoglycerate (BPG)

(d) Findings on palpation, percussion, auscultation


❖ Palpation: reduced chest wall movement, trachea centrally located
❖ Percussion: dullness to percussion
❖ Auscultation: bronchial breath sounds, abnormal vocal resonance
(increased), crackles

6. What is the motor root value of the following:


Biceps-C5,6
Triceps-C6,7,8
Knee-L3,4
Ankle-S1,2

39 | P a g e ...TROY MADAH KAMPIRA...


2019 END OF YEAR PAEDIATRICS SAQs

1) A four-year-old boy presents to A & E. His mother states he thinks he is


having the “same thing he had last year.” You look in his health passport
and see he was diagnosed with nephrotic syndrome at that time and
treated successfully with prednisone
a) List two symptoms that would be consistence with the diagnosis of
nephrotic syndrome (2marks)
1) Peri-orbital oedema as well as peripheral pitting oedema(anasarca)
2) Abdominal pain and diarrhea with ascites
3) Respiratory distress due to massive ascites
compressing the diaphragm4) Anorexia, irritability, fatigue

b) Describe 4 laboratory tests abnormalities seen in nephrotic syndrome


(4marks)
1) Heavy proteinuria of >3.5g/dl in a 24hr urinary protein excretion
assessment. This is done after a persistent significant proteinuria of 3+ or 4+
detected
by a dipstick
2) Serum albumin levels of <2.5g/dl
3) Elevated serum lipids where there is increased LDL and VLDL
4) Presence of granular and fatty casts in urine microscopy

c) Name 2 complications of nephrotic syndrome and explain why each


occurs (4marks)
i. Infections
❖Urinary immunoglobulin losses as well as complement factor (properdin
factor B) makes the body susceptible to bacteremia and peritonitis
particularly from
Streptococcus pneumoniae and Escherichia coli
❖The use of immunosuppressive medications to treat nephrotic syndrome
40 | P a g e ...TROY MADAH KAMPIRA...
ii.Thrombosis
i. A hypercoagulable state due to urinary losses of antithrombin,
thrombocytosis which may be exacerbated by steroid therapy, increased
synthesis of clotting factors and increased blood viscosity from the raised
hematocrit predisposes to thrombosis

d. Describe any two complications of nephritic syndrome (2 marks)


i. Acute renal dysfunction
ii. Hypertension
iii. hypertensive encephalopathy
iv. heart failure
v. Seizures
vi. Hyperkalemia
vii. Hyperphosphatemia
viii. hypocalcemia
ix. Acidosis

2. A. State any two causes of stridor in children (2marks)


I. Acute epiglottitis
II. Diphtheria
III. Infectious mononucleosis
IV. Inhaled foreign body
V. croup
B. Complete the tale below
Pathological Chest wall Tracheal Percussion Breath Vocal Added
process Movement displacement note sounds resonance sounds

41 | P a g e ...TROY MADAH KAMPIRA...


Normal Symmetrically Centrally Resonant Vesicular normal No
located or added
Tympanic sound

Asthma Centrally Normal Vesicular normal No


located prolonged added
sound

Consolidation Affected side None Dull Decreased increased


(Bronchial)
Pleural Affected side Away from Stony dull Attenuated Decreased
effusion the diseased or absent or absent
side

Pneumothorax Affected side Away from Hyper- Distant or Decreased None


diseased side resonance absent or absent
(8marks)

3)There was a diagram of cardiac cycle.


The diagram showed the pressure measured in the left ventricle (LV), right
ventricle(RV), right atrium(RA), left atrium(LA) and aorta throughout the cardiac
cycle
– With reference to specific cardiac valves, describe what happens at point: A, B,
C, D (4marks)

❖Point A:
❖Point B:
❖Point C:
❖Point D:
A 15-year-old girl develops mitral regurgitation as a result of rheumatic heart
disease. Between which 2 points on the diagram above will the murmur be
heard? (1mark)
42 | P a g e ...TROY MADAH KAMPIRA...
List 3 characteristics that are typical of mitral regurgitation murmur (3marks)
1) Pansystolic murmur
2) Loudest at the apex
3) Radiates to the axilla or base of the heart
4) Its blowing and high pitched
5) Increased during expiration and reduced during inspiration

Briefly describe the pathogenesis of rheumatic heart disease (2marks)


❖Following an acute attack of streptococcal pharyngitis by group A beta
hemolytic Streptococci. Within 2-4 weeks after this attack anti streptococcal
antibodies form and attack the heart and other extra cardiac sites.
Antibodies cross react with antigens of cardiac sarcolemma and glycopeptides
causing myocarditis and valvulitis, hence rheumatic heart disease. Vegetation,
Aschoff bodies form in the myocardium and uncommonly in the valves. Later on,
they heal by fibrosis causing valve stenosis or insufficiency.

• Following an acute attack of streptococcal pharyngitis by group A beta


haemolytic streptococcus, rheumatic fever can develop. The body then forms
Anti streptococcal antibodies which attack the heart and cause pan-carditis,
myocarditis and endocarditis and this causes insufficiency and damage to the
valves especially the mitral valve. The valves heal by fibrosis and scarring hence
rheumatic heart disease

2019 END OF YEAR DIFFERED PAEDIATRICS SAQs

QUESTION 7: SICKLE CELL DISEASE


a) List 5 indications of blood transfusion in sickle cell disease (5m)

43 | P a g e ...TROY MADAH KAMPIRA...


• Symptomatic anaemia: Aplastic crisis, Acute Splenic
sequestration
• Acute hepatic sequestration/ intrahepatic cholestasis
• Acute Chest Syndrome (ACS)
• Acute multiorgan failure
• Preoperative (Surgeries lasting >1hr and require general
anesthesia)
• Pregnancy

b) What would be your advice to the mother to prevent sickle cell crisis? (3m)
• Tell the mother to give enough water and breast feed the child
adequately to prevent dehydration which triggers sickling of RBCs
• Tell her to control hypothermia by insulating the baby adequately
since hypothermia also triggers sickling of RBCs
• The mother should not let the child to do very intense exercises
which leads to hypoxia that triggers sickling of RBCs

c) How does hydroxyurea decrease the frequency of sickle cell crises? (2m)
• Stimulates the production of fetal haemoglobin that have high affinity
for oxygen thereby preventing hypoxia that triggers RBCs sickling
• It increases surface area of red blood cell by making them bigger,
rounder and flexible there by reducing sickling of the RBCs
• It helps in hydrating RBCs thereby reducing dehydration that triggers
sickling of RBCS
• It increases production of Nitric Oxide (NO2) to stimulate
vasodilation that prevents RBCs sickling

QUESTION 8.
Andrew, a 3-year-old boy was brought to a clinic because he had been sick with
diarrhea for the last 3 days. He well nourished, and low-grade fever, if this boy

44 | P a g e ...TROY MADAH KAMPIRA...


was having severe dehydration which signs would he have? (10marks)
(2.5marks for each correct answer)
i. Lethargy/ unconsciousness
ii. Sunken eyes
iii. Unable to drink/ drink poorly
iv. Skin pinch goes back very slowly (>2 seconds)

QUESTION 9: DKA
A 6-year-old is admitted to the hospital with a diagnosis of DKA. On physical
examination he is found to have fast and deep breathing and is severely
dehydrated.
a) From the history how would differentiate the dehydration in DKA
and dehydration in severe gastro-enteritis? (3m)
i. Thirst
ii. Polyuria
iii. Polydipsia
iv. Polyphagia

b) Explain why this child has deep breathing? (3m)


Metabolic acidosis

c) In addition to insulin and possible treatment for infections, what


other treatment would you consider? (2m)
i. Careful replacements of fluid deficits
ii. Correction of electrolyte imbalances
iii. Monitoring for complications of treatment

d) If the child starts seizing during the treatment, what could be the
possible cause? (1m)
Cerebral oedema

45 | P a g e ...TROY MADAH KAMPIRA...


2018 END OF YEAR PAEDIATRICS SAQS

Same as questions in 2020 End of year Paediatrics question above

2018 END OF YEAR SUPPLIMENTARY EXAM- PAEDIATRICS QTNs

1. Which of the following lists of clinical signs is most in keeping with a left
pneumothorax?
Option Air entry Percussion note Apex beat
deviation
A Normal Dull No deviation
B Normal Normal To right
C Reduced Hyper-resonant To right
D Reduced Hyper-resonant To left
E Reduced Stony dull To left

2. Which of the following is the best definition of bronchial breathing?


a. Equal length inspiratory and expiratory phase separated by a short
gap; whispered words heard clearly over lung fields
b. Equal length inspiratory and expiratory phase, separated by a short
gap; crackles heard in expiration
c. Inspiratory phase longer than expiratory phase, no gap between
them. No added sounds.
d. Prolonged expiratory phase, no gap between inspiration and
expiration, high pitched noise in expiration
e. Prolonged inspiratory phase, short gap between inspiration and
expiration; harsh noise during inspiration
46 | P a g e ...TROY MADAH KAMPIRA...
3. Which of the following is NOT an Emergency Treatment and Triage (ETAT)
respiratory emergency sign?
a. Absent breathing
b. Cyanosis
c. Loud wheezing
d. Obstructed breathing
e. Severe respiratory distress

4. A 10-month-old baby presents to hospital with respiratory distress. She has


had an upper respiratory tract infection for 2 days. On examination there is
intercostal recession, tachypnoea, an audible wheeze and fine crackles
throughout the lung fields. The organism most likely to cause this illness is:
a. Bordetella pertussis
b. Pneumocystis jiroveci
c. Respiratory syncytial virus
d. Rhinovirus
e. Streptococcus pneumonia
5. When resuscitating adults, the correct ratio of chest compressions to inflation
breaths is:
A 3:1
B. 5:1
C. 15:2
D. 30:2
E. 30:1

6. According to WHO guidelines, the upper limit of normal for the respiratory
rate of a child who is 3 months old is:
A. 80 breaths per minute (BPM)
B. 60 BPM
47 | P a g e ...TROY MADAH KAMPIRA...
C. 50 BPM
D. 40 BPM
E. 30 BPM

7. According to the WHO guidelines, the most appropriate first-line antibiotic for
the treatment of severe pneumonia in children is:
a. Amoxycillin
b. Benzyl penicillin
c. Ampicillin and gentamicin
d. Chloramphenicol
e. Ceftriaxone

8.The typical clinical signs (on percussion & auscultation) of a pleural effusion are:

A. Dullness to percussion, bronchial breathing, increased vocal resonance


B. Dullness to percussion, loud crepitations, reduced vocal resonance
C. Dullness to percussion, reduced or absent breath sounds, reduced vocal
resonance
D. Hyper-resonance on percussion, absent breath sounds, reduced vocal
resonance
E. Hyper-resonance on percussion, vesicular breath sounds, normal vocal
resonance

9. A 1-year-old child presents to the paediatric accident and emergency


department with a 2-day history of vomiting and diarrhoea. She has
vomited three times today and there is no blood or bile in the vomit. She
has passed 5 loose watery stools today. There is no blood or mucous in her
stools. On examination she is breathing normally, has no signs of
respiratory distress, has warm hands and sunken eyes.
48 | P a g e ...TROY MADAH KAMPIRA...
Which one of the following additional findings would lead you to classify her as
having severe dehydration rather than some dehydration?

A. Lethargy
B. Drinking eagerly
C. Dry mucous membranes
D. Poor urine output
E. Capillary refill 1 second

10.Grace is a 4-year-old who attends A & E following profuse watery diarrhoea


for 5 days and vomiting for 2 days. Her clinical findings are of lethargy, heart
rate of 150 beats per minute, respiratory rate of 35/minute, capillary refill of 4
seconds and cold peripheries. The following statements are true.

A. The heart rate is within the acceptable normal range for this child.
B. A capillary refill is measured by holding a toe or finger for 3 seconds, letting
go and seeing if perfusion returns within 2 seconds.
C. A capillary refill time of 4 seconds is normal
D. The above findings suggest that this child is shocked
E. The child has gastroenteritis with moderate dehydration.

11. A 4-year-old girl presents with severe vomiting of non-bilious in nature for 2
days. Which of the following blood gas findings is consistent with this clinical
presentation?

A. Hypochloremic, hypokalemic metabolic alkalosis


B. Hypochloremic, hyperkalemic metabolic alkalosis
C. Hypochloremic, hypokalemic metabolic acidosis
D. Hypochloremic, hyperkalemic metabolic acidosis
E. Hyperchloremic, hyperkalemic metabolic alkalosis

12.The most common cause of diarrhoea in children is:

49 | P a g e ...TROY MADAH KAMPIRA...


A. Shigella
B. Vibrio Cholera
C. Rotavirus
D. Giardia
E. E. coli

13. A ten-year-old boy, who is HIV infected, has enlargement of the salivary
glands, digital clubbing, and generalized lymphadenopathy. An x-ray reveals a
reticular pattern.

What is the most likely diagnosis?


A. Tuberculosis
B. Bronchiectasis
C. Pneumocystis jiroveci pneumonia (PCP)
D. Lymphocytic interstitial pneumonitis (LIP)
E. Cytomegalovirus (CMV) pneumonitis
14. An eleven-year-old girl is known to have rheumatic heart disease with mitral
regurgitation. Where would you expect the murmur to be heard loudest?

A. At the back
B. Cardiac apex
C. Left lower sternal edge
D. Left upper sternal edge
E. Right upper sternal edge

15. A four-year-old boy presents to the paediatric accident and emergency with a
two-year history of cough and shortness of breath on exertion. On
examination he has finger clubbing and cyanosis. He is not in respiratory
distress. His apex beat is in the fifth left intercostal space, mid clavicular line.
There are no heaves or thrills. He has normal 1st and 2nd heart sounds with a
grade 3/5 ejection systolic murmur heard loudest at the left upper sternal

50 | P a g e ...TROY MADAH KAMPIRA...


edge, which radiates to the back. What is the most likely explanation for these
physical findings?

A. Ventriculo-septal defect
B. Rheumatic Heart Disease
C. Transposition of the great arteries
D. Tetralogy of Fallot
E. Patent Ductus Arteriosus

16. You auscultate the heart of a one-year-old boy. You hear a normal first and
second heart sound. The first heart sound is caused by:

A. Closing of the Tricuspid and Mitral Valve


B. Opening of the Tricuspid and Mitral Valve
C. Closing of the Aortic and Mitral valve
D. Opening of the Pulmonary and Aortic Valve
E. Closing of the Pulmonary and Aortic Valve

17.A ten-year-old boy with Sickle Cell disease is described as having upper motor
neurone signs following a stroke affecting the right side of his body. He has a
right sided facial weakness. Which of the following would help you distinguish
a facial weakness caused by an upper motor neurone lesion rather than a
lower motor neurone lesion?

A. Decreased tear production


B. Mouth deviating to normal side
C. Loss of taste sensation to anterior two thirds of the tongue
D. Loss of corneal reflex
E. Bilateral forehead wrinkling when asked to raise eye brows

18.A four-year-old HIV uninfected boy presents with seizures and headache. He
has a Blantyre coma score of 4/5 on presentation. A lumbar puncture is taken
51 | P a g e ...TROY MADAH KAMPIRA...
and the CSF is cloudy. The CSF subsequently grows Streptococcus Pneumoniae.
Which of the following long-term complications is most likely to occur?

A. Hydrocephalus
B. Cerebral palsy
C. Hearing impairment
D. Visual impairment
E. Epilepsy

19.The Blantyre Coma Score of a child who localizes to pain, produces an


inappropriate cry and has wide open eyes but does not follow an object is:

A. 0/5
B. 1/5
C. 2/5
D. 3/5
E. 4/5

20. Children with measles frequently have a characteristic rash which can be
described as:
A. Papulovesicular
B. Urticarial
C. Maculopapular
D. Petechial
E. Nodular
21. Which of the following vaccines on the Malawi infant vaccination schedule is
a live vaccine?

A. Diphtheria
B. Haemophilus influenza type b
C. Hepatitis B
D. Bacillus Calmette-Guerin (BCG)
E. Tetanus
52 | P a g e ...TROY MADAH KAMPIRA...
22. BCG vaccine is given to new-borns in Malawi as part of the routine
immunization schedule. Which of the following is an effect of BCG which
leads to its recommendation by WHO?

A. It reduces primary TB infection


B. It reduces pulmonary TB
C. It reduces TB transmission
D. It reduces TB meningitis
E. It reduces TB adenitis
23. A Healthy term new-born baby, with normal development would be
expected to:

A. Lie supine in a frog leg posture


B. Fix and follow a face through 180 degrees
C. Have head lag when pulled up from lying
D. Make sounds like “ah” and “eh”
E. Turn their head towards a voice
24. A nine-month-old with normal development would be expected to:

A. Point at objects on request


B. Feed with a fork
C. Put two words together
D. Sit without support
E. Scribble

25. From what age would you consider a positive HIV ELISA test in a child to
be diagnostic of HIV infection?

A. Birth
B. 6 months
C. 12 months
D. 18 months
E. 24 months
53 | P a g e ...TROY MADAH KAMPIRA...
26. Neonates born with the following weight are considered as Low Birth
Weight

A <4000g
B <3500g
C *<2500g
D Between 2500g and 3000g
E <1500g

27. An infant whose urine on dipstick shows +protein, + blood,


+++leukocytes and +++ nitrites and negative microscopy probably has got

A Bilharzia
B Nephrotic syndrome
C Glomerulonephritis
D Urinary tract infection
E Salpingitis

Questions 28 – 30; Extended matching


From the following causes of breathlessness in children, select the correct cause
for each scenario.
a. Acidosis
b. Anaemia
c. Asthma
d. Bronchiectasis
e. Bronchiolitis
f. Cardiac failure
g. Pleural effusion
h. Pneumococcal pneumonia
i. Pneumocystis jiroveci pneumonia
j. Pneumothorax
54 | P a g e ...TROY MADAH KAMPIRA...
28. A 4-year-old girl with fever (39oC), right sided pleuritic chest pain. On
examination, she has herpes labialis. Respiratory rate = 42 breaths per minute
with subcostal recession. There is dullness to percussion at the right base with
bronchial breathing.
ANSWER: H
29.A 9-year-old boy with a history of vomiting and increasing confusion. On
examination he is significantly dehydrated. His peripheries are cool and capillary
refill time = 4 seconds. Respiratory rate = 51 breaths per minute with deep
sighing respirations. There is no recession or use of accessory muscles. Breath
sounds are vesicular with no added sounds. Oxygen saturation in air = 93%.
ANSWER: A
30. A 7-month-old baby presents with poor feeding. On examination he is
coryzal and mildly febrile (37.8oC). Respiratory rate = 55 breaths per minute
with an audible wheeze. There is sternal, intercostal and subcostal recession. On
auscultation, the expiratory phase of respiration is prolonged. There are fine
crackles and an end-expiratory wheeze heard throughout the lung fields. Oxygen
saturation in air = 85%.
ANSWER: E

SHORT ANSWER QUESTIONS

31.Fill in table below the difference in the cerebrospinal fluid between viral
meningitis and bacterial meningitis (10 marks)

CSF Parameter Normal Bacterial meningitis Viral meningitis

55 | P a g e ...TROY MADAH KAMPIRA...


WBC (mm3) 0 neutrophil High; 100 - 50,000 Slightly raised; 25-
500
0-6 With neutrophils
Lymphocytes dominant With Lymphocytes
dominant
Opening pressure Normal High Normal or increased
Protein 0.15 - Mild to moderately Mildly increased
0.45mg/L increased
Glucose 3.6 – 5.6 Low; 1.1 - 1.6 Within normal range
mmol/L mmol/L
Appearance Clear Cloudy Clear

32. a) List the four major developmental domains (4 marks)

I. Gross motor
II. Fine motor and Vision
III. Hearing and Speech
IV. Social
b) For each of the four developmental domains give one example of what
you would expect a one-year-old child with normal development to be
able to do in that domain. (4 marks)
Gross Motor: ❖ Walking
Fine Motor & Vision: ❖ Pincer grip ❖ Looks at pictures
Hearing & Speech: ❖ Says first word ❖ Responds to own name
Social: ❖ Drinks from cup ❖ waves

c) List four common causes of developmental delay in Malawi (2 marks)


I. Perinatal asphyxia
II. Meningitis
III. Cerebral Malaria
56 | P a g e ...TROY MADAH KAMPIRA...
IV. Down’s syndrome
V. Cerebral palsy
VI. Chronic disease (HIV, TB)
VII. Malnutrition
33. a) List five clinical signs of kwashiorkor (5 marks)

1) Failure to thrive
2) Oedema
3) Anorexia
4) Diarrhoea
5) Skin and mucous membrane lesions
6) Misery or apathy

b) The WHO has listed ten steps of management for severe malnutrition
which provide principles for the management of severe malnutrition.
List five of these ten steps. (5 marks)
(I)Prevent and treat hypoglycemia;
Rationale: Feed with f-75 or 10% glucose, if unconscious treat with IV 10%
glucose 5ml/kg.

(ll)Prevent and treat hypothermia,


Rationale: Feed, wrap the baby, KMC and use heater.

(lll)Prevent and treat dehydration,


Rationale: ReSoMal rehydration fluid orally or by NGT

(IV)Prevent and treat infections,


Rationale: Broad-spectrum antibiotics [amoxyl if in shock x-pen] give one
dose of measles vaccine on discharge to all children over 9mos & albendazole

57 | P a g e ...TROY MADAH KAMPIRA...


(V)Correct electrolyte imbalance;
Rationale: Therapeutic milk and ReSoMal

(VI)Correct micronutrient deficiencies,


Rationale: Folic acid 5mg stat and vit A orally day 1,2 &14[<6mos:50,000iu;6-
12mos:100,000iu & older give 200,000]

(VII) Start feeding cautiously,


Rationale: Frequent feeding every 2-3 hrs. oral small feeds of low osmolality and
low lactose & also consider f-75

(VIII)Give catch-up diet,


Rationale: Make a gradual transition from starter f-75 to catch up formula f-100
or ready-to-use-therapeutic food [RUTF] over 2-3 days

(IX)Provide tender loving care, play & stimulation,


Rationale: TLC, cheerful, stimulating environment, structured play therapy[15-
30min/day] &p/activity as soon as child is well

(X)Prepare for discharge and follow up.


Rationale: Before discharge, the infant’s vaccination status and other routine
interventions should be checked and provided as appropriately. Mothers or
caregivers should then be linked with any necessary community follow up and
support. Effective BF, adequate wt gain, WFL equal or > -2z scores

2018 END OF PAEDIATRICS ROTATION QTNs


1. How can you calculate the w/h percentage?

58 | P a g e ...TROY MADAH KAMPIRA...


2. Define Cerebral malaria (Definition according to WHO)
Malaria due to plasmodium falciparum (+ MPs) with reduced levels
of consciousness which is 2/5 BCS or less when all other causes have
been excluded.

3. Fluid management in malnourished child with shock


a. If patient is able to drink or can tolerate NGT
i. ReSoMal 5 ml/kg every 30 minutes for 2 Hours,
ii. then 5-10 ml/kg/hour for 4 to 10 hours
iii. if rehydration continues after 10 hours start F75

b. If patient is unconscious or vomiting all feed


i. ½ strength Darrow’s 15 ml/kg + 5% Dextrose or LR 15 ml/kg +
5% Dextrose or 0.9 saline + 5% dextrose for 1 hour

ii. Observe HR and RR for the hour, if improving then its


dehydration,
- give another 15 ml/kg for another hour
- start oral fluids and F75 at 10 ml/kg/hour for 10 hours

iii. If HR and RR not improving then cause is sepsis


and not dehydration
- Give blood 10 ml/kg for 3 hours

4. What are diagnostic tests used in malnourished child and TB?


a. Chest x-ray
b. Mantoux test of induration 5 mm or more
5. (There was a case scenario). What is underlying cause?
Immunosuppression by the malnutrition

59 | P a g e ...TROY MADAH KAMPIRA...


6. What are the complications of condition above?
a. Pericardial effusion
b. Pleural effusion
c. TB meningitis
d. Miliary TB to sites such as bones, joints and kidneys
e. Intestinal TB
f. TB pneumonia
g. Empyema
h. Necrosis, fibrosis and cavitation of the lung tissue

7. What are the characteristics of simple febrile seizure?


a. 6 months to 5 years
b. Less than 15 minutes
c. Its familial
d. No neurological defect
e. It is generalized
f. Does not occur in series or repeat in the same illness
g. Postictal phase is absent or brief
h. High fever (>38.5)

8. What are the characteristics of Complex febrile seizure?


a. Prolonged duration
b. Transient neurological defect
c. Occurs in series and repeated in the same illness
d. It is focal
e. Not familiar
f. Has chance of developing epilepsy later

9. Give 4 reasons for severe malaria and respiratory distress

60 | P a g e ...TROY MADAH KAMPIRA...


a. Severe anemia due to hemolysis, impaired erythropoiesis and
removal of damaged red cells by the spleen- reduce hemoglobin and
O2 carrying capacity.
b. Non cardiogenic pulmonary edema- reduced o2 diffusion between
alveoli and capillaries.
c. Circulatory collapse due to bacteremia (co-infection with NT
salmonella)
d. Metabolic acidosis- compensatory fast breathing
e. Cerebral malaria (reduced level of consciousness, secretive seizures) –
sequestration of parasitized red blood cells
f. Concomitant pneumonia

10. Mention two causes of infections in nephrotic syndrome


a. Loss of immunoglobulins, complement factor C3b and opsonins in
urine
b. The use of immunosuppressive drugs in nephrotic syndrome

11. UTI and posterior urethral valve


a. Posterior urethral valve causes obstruction of the posterior urethra
causing bilateral hydronephrosis. The obstruction (stasis) increases
concentration of microbes which can cause UTI

12. Mention 5 causes / risk factors for hypothermia in neonates


a. Large surface area relative to the body
b. Immature liver for gluconeogenesis
c. Thin skin
d. Lack of subcutaneous fat
e. Neonatal sepsis
f. Lack of response (i.e., shivering)

61 | P a g e ...TROY MADAH KAMPIRA...


13. Eisenmenger’s Syndrome
a. R to L shunting through untreated VSD or ASD secondary to
irreversibly pulmonary hypertension
b. Secondary to irreversible damage to the pulmonary vasculature after
chronic volume and pressure overload

14. PMTCT, & components of option B+


a. Provider initiated early HIV testing for early diagnosis and ART
initiation
b. Initiation of life-long ART for all HIV infected pregnant and
breastfeeding women c. Safe obstetric practices
ii Provision of infant nevirapine prophylaxis
iii Exclusive breastfeeding in the first 6 months

15. Mention two investigations used is a suspected Intussusception patient


a. Abdominal Ultrasound scan
b. Abdominal X-ray and barium enema

16. Explain the Pathophysiology of finger clubbing


a. Chronic hypoxia leads to vasodilatation in the fingers, leading
increased blood supply, and then high proliferation of nail bed tissue
due to growth factors secretion
17. At what age is Rubella vaccine given?
a. 9 months first dose
b. 15 months 2nd dose

18. Explain the pathophysiology of hypertension in nephritic syndrome

19. Mention one TB prophylaxis


a. Isoniazid Preventive Therapy- prevents progression of disease
62 | P a g e ...TROY MADAH KAMPIRA...
2017 END OF YEAR PAEDIATRICS SAQs

63 | P a g e ...TROY MADAH KAMPIRA...


64 | P a g e ...TROY MADAH KAMPIRA...
ANSWERS TO QUESTIONS ABOVE
QUESTION 7
What is the most likely diagnosis
• acute poststreptococcal glomerulonephritis

What is the mechanism of the oedema


• There is urinary loss of proteins which leads to hypoalbuminemia and this
reduces the plasma oncotic pressure? As a result, there is transudation of
fluid from the intravascular compartment leading to a decrease in
intravascular volume. A decrease in intravascular volume causes a fall of
the renal perfusion pressure and this activates the renin-angiotensinaldosterone
system which stimulates the tubular reabsorption of sodium
thereby causing oedema.

What investigations would you do to confirm diagnosis?


• Urine dipstick
• Urine microscopy
• Urinalysis
• Renal ultrasound scan

What further laboratory investigations would you do?


• U&Es
• FBC

What is the management for this condition?


• Diuresis with systemic furosemide
• 10-day course of systemic antibiotics e.g., penicillin
• Calcium channel antagonists
• Sodium restriction
65 | P a g e ...TROY MADAH KAMPIRA...
• Renal dialysis

What are complications?


• Hypertensive encephalopathy
• Intracranial bleeding
• Acute renal failure
• Heart failure
• hyperkalemia
What are predisposing factors?
i. Streptococcal throat infection
ii. Hemolytic uremic syndrome
iii. Henoch scholnein purpura
iv. Alport’s syndrome
v. IgA nephropathy

QUESTION 8
What would be the initial management of Chisomo?
• Make sure the airway is patent, that there is adequate respiratory effort and
that there is adequate circulatory perfusion.
• Put child in decubitus position
• Check blood glucose, if hypoglycemia give dextroseD10% 5ml/kg IV
• If no hypoglycemia give diazepam 0.15-0.25mg/kg IV or 0.5mg/kg rectally
Which investigations would you do?
• RBS
• U&Es
• LFT
• LP
• MRDT
• Blood culture

66 | P a g e ...TROY MADAH KAMPIRA...


• HTC
• MPs
• FBC
• CHEST X-RAY

QUESTION 9
a. Floppy baby (Atonic Cerebral Palsy)

b. Possible causes
i. Neonatal meningitis
ii. Birth asphyxia
iii. Polio
iv. Kernicterus

c. Common problems
i. Head pathologies: microcephaly, or macrocephaly due to
hydrocephalus)
ii. Visual pathologies: distorted vision, visual fields and ocular
movement (e.g., myopia, squint).
iii. Ear pathologies: Sensorineural deafness and chronic serous otitis
media)
iv. Bulbar palsy
v. Back pathologies: Kyphoscoliosis
vi. Chest pathologies: Chest infection
vii. Abdomen pathologies: Constipation
viii. Hip pathologies: Dislocation

67 | P a g e ...TROY MADAH KAMPIRA...


ix. Malnutrition
x. Teeth pathologies: Dental carries

d. Management is multifaceted & complex comprising of a multidisciplinary


team of:
i. Occupational therapist iv. Orthopedic surgeon
ii. Physiotherapist v. Psychologist
iii. Speech therapist

2017 END OF 4TH PAEDIATRICS ROTATION QTNs


1) Describe the Eisenmenger syndrome
a R to L shunting through untreated VSD or ASD secondary to
irreversibly pulmonary hypertension
b Secondary to irreversible damage to the pulmonary
vasculature after chronic volume and pressure overload

2) Child presenting with brisk left reflexes and reduced left tone has:
Some of the Options given were:
A. Right Hemiplegia
B. Left Hemiplegia
C. Spastic Quadriplegia

3) i. Know the dosage of ReSoMal to give in a malnourished


ii. know the dosage for shock and dehydration child

68 | P a g e ...TROY MADAH KAMPIRA...


4). Know the Likely causing of Hydrocephalus in a child of some months.
Some of the Options given were:
A. Intraventricular hemorrhage

B Posterior fossa Neoplasm


C Aqueduct stenosis

5). Know Laryngo-tracheal-bronchitis (croup) causative organism. Some of the


Options given were:
i Parainfluenza
ii RSV
iii Haemophilus Influenzae

6). Know the Treatment of Chronic Otitis Media. Some of the Options given:

A. IV antibiotic
B. Ear Droplets

7). Which is the least likely to be found in Kwashiorkor. Some of the Options
given were:
A. . Apathy
B. . Hepatomegaly

C. . Ascites

D. . Skin Lesions
69 | P a g e ...TROY MADAH KAMPIRA...
8). Which Perinatal Condition is helped by Maternal Corticosteroids?
Some of the Options given were:
A. Respiratory Distress Syndrome
B. Transient Tachypnea of the Newborn

9). Which Neonatal condition is common in babies born by Caesarian Section?


• Transient Tachypnea of The Newborn

10). Most likely to cause an epidemic of Meningitis. Some of the Options given
A. Neisseria Meningitidis
B. Haemophilus Influenzae
C. Group B Streptococcus

11). Most likely to Cause Neonatal Meningitis. Some of the Options given
A. Group B Streptococcus
B. Haemophilus Influenza

12). Commonest cause of Meningitis in Malawi. Some of the Options given were:
A. Streptococcus Pneumonia
B. Haemophilus Influenzae
13). What is the presentation of Neonatal Meningitis?

70 | P a g e ...TROY MADAH KAMPIRA...


• A. Bulging fontanelle
14). Which Neonatal condition is associated with continuous Positive Airway
Pressure?
Pneumothorax
15). Which Neonatal condition is characterized by Air trapping?

Meconium Aspiration (Bohr valve effect)


16). Which neonatal condition is caused by premature rupture of membranes?
Neonatal sepsis

18). What is the characteristic difference between oedema caused by malnutrition


and that caused by Nephrotic Syndrome?

71 | P a g e ...TROY MADAH KAMPIRA...


19). A Child who had been passing loose stools and then vomiting what kind of
dehydration does he have some of the Options given
A. Hypernatremia
B. Hyponatremic
C. Hyperkalemic
D. Hypokalemic

19). A Child with Moderate Acute Malnutrition what are the parameters?

20). Know the Respiratory condition presentation of the respiratory conditions


that are part of the vaccination

21). 9-month-old child who presented with painful swelling


Pyloric Stenosis Presentation

22). Child has abdominal pain curls up


Intussusception

23). TB Potts in the Lumbar Region. Will it cause an upper or a Lower motor
neuron lesion?
Upper motor neuron lesion
24). Child presenting with Red Eyes. Which nutrient is deficient in her body?
Vitamin A
25). What Receptor does Salbutamol stimulate?

72 | P a g e ...TROY MADAH KAMPIRA...


Beta Receptor 2
26). Child presented with central abdominal pain
27). Management of a child with ALL who is known to be neutropenic and
Temperature above 38.5 degrees Celsius. How do you manage him?
Some of the Options given were:
A. FBC and inflammatory Test
B. Admit and observe
C. Start Antibiotics immediately

28). What is the most likely Complication of someone presenting with signs of
Meningitis?
Hearing impairment
29). A history of a child with an MCV of 60. What is the treatment?
Some of the Options given were:
A. Iron supplement
B. Folic Acid supplement
C. Vitamin B 12

30) Picture of a lung with tachypnea, hyperinflated chest, bilateral wheezes, fine
crackles, Recessions. What is the diagnosis? Some of the Options given were:
A. Pneumonia
B. Bronchiolitis

73 | P a g e ...TROY MADAH KAMPIRA...


31…...A Child presented with central abdominal pain is slightly pale and 3cm
splenomegaly. What is the likely diagnosis? Some of the option given were:
A. Hookworm infestation
B. SCD
C. Malaria
D. ALL

32. A child with a boot shaped heart on a chest x-ray. What is the
diagnosis?
Tetralogy of Fallot

74 | P a g e ...TROY MADAH KAMPIRA...


2016 END OF YEAR PAEDIATRICS SAQs

75 | P a g e ...TROY MADAH KAMPIRA...


ANSWERS TO QUESTIONS ABOVE
QUESTION 9
Explain how RESOMAL is different from ORS and why this is relevant in a
malnourished child in the ward
• ReSoMal has less sodium, more potassium and glucose than ORS. It is relevant
for the malnourished child because they have a lot of sodium and hypokalemia in
their bodies.

Explain the pathogenesis of diarrhea in malnutrition


• Malnutrition causes atrophy of the villi and epithelial cells of the bowel mucosa
hence the absorption of water and nutrients is reduced and this leads to diarrhea.
• Malnutrition also reduces immunity which predisposes to different infections
that may cause diarrhea.

76 | P a g e ...TROY MADAH KAMPIRA...


Explain how HIV may cause malnutrition
HIV causes increased energy expenditure- there is increase in resting energy
expenditure (or the resting metabolic rate/RMR). Children require a higher
proportion of energy for growth and development. HIV causes malabsorption of
nutrients from the intestines- food may be poorly absorbed in patients with HIV
and AIDS. Intestinal malabsorption and nutrient loss are common. The virus has
been shown to damage the intestinal villi, and inflammation can damage gut
tissue and reduce absorption. These changes in the gut seem to affect the body’s
ability to utilize dietary fat and carbohydrates.

• People with HIV and AIDS often eat less — most commonly because of a loss
of appetite. In addition to underlying HIV, a number of opportunistic infections
contribute to this by causing nausea, malaise, and fever. Infections, such as
esophageal candidiasis, that cause a sore mouth or pain from eating, also decrease
food intake — and this may occur silently in children

• “HIV seems to induce a special metabolic effect involving a preferential loss of


protein over fat
Discuss different non-medical risk factors of malnutrition
• Not being breastfed in poor social circumstances
• Being a twin
• Having a father who is alcoholic or out of job
• Has lost a parent by death or desertion
• Has a working mother and there is inadequate child minding

2015 END OF PAEDIATRICS ROTATION SAQs


Areas in SAQs (Same as in 2019 End of year Paediatric already written above)

1. Nephrotic syndrome

77 | P a g e ...TROY MADAH KAMPIRA...


2. Filling the table of respiration conditions
3. Cardiac cycle

Areas in matching questions


1. Measles, hepatitis....
2. Respiratory cases, pneumonia, asthma……they were asked to identify
causing bugs or the actual disease
3. Cranial nerves: optic nerve, oculomotor, facial nerve

Areas in MCQs
1. Maintenance fluid of a 32 kg patient
2. DKA management......first thing to do
3. CSF quality and related bugs
4. Worldwide cause of watery diarrheal deaths
5. Cause of a watery diarrhea outbreak in a small community
6. The immunization schedule and diseases that are to be prevented
7. Malnutrition
8. UTI (Causative agent: gram negative cocci)
NB: Master the diagrams. Most of the questions were based on these.
e.g., How do children with whooping cough look like?

2014 END OF 1ST PAEDIATRIC ROTATION

1. Development milestone question repeated as in 2019 End of year rotation


above.

2. An 18-month-old child is admitted because of cough and fever. On


examination, the doctor recognizes tachypnea, intercostal and sternal

78 | P a g e ...TROY MADAH KAMPIRA...


recessions, and bronchial breathing on the left lung. The diagnosis of
pneumonia is made.
a. Define bronchial breathing. (3 marks)

Breath sounds of a harsh or blowing quality, heard on auscultation of


the chest made by air moving in the large bronchia and barely if at all,
modified by the intervening lung

Hollow blowing sound heard due to increased density of matter in the


peripheral lung.

b. What is the upper limit of respiratory rate for a child of this age?
(1 mark)
40 breaths per minute

c. The oxygen saturation is low. The doctor examining the child knows
that the oxygen dissociation curve will be shifted to right. List three
factors that will shift this curve to the right? (3 marks)
i. ↑ H+ (Bohr shift)
ii. ↑ temperature
iii. ↑ p(CO2)
iv. ↑ 2, 3-Biphosphoglycerate (BPG)

v. Suggest three other clinical signs that you would expect to find on
examination of this child’s respiratory system on the left side. (3
marks)

(i)Palpation: ❖ Trachea centrally located


(ii)Percussion: ❖ Dullness
(iii)Auscultation: ❖ Reduced or bronchial breath sounds, ❖
Abnormal vocal resonance(increased), ❖ Crackles
79 | P a g e ...TROY MADAH KAMPIRA...
2014 END OF PAEDIATRICS ROTATION QTNs
1. There was a case of microcytic hypochromic anemia in a child
a. What is the commonest cause of the condition in Malawian children?
Iron deficiency
b. Mention other rare causes of the condition?
▪ Lead poisoning
▪ Sideroblastic anemia,
▪ Sickle cell disease
▪ Thalassemia
▪ ??Chronic disease, PUD, TB, HIV, Malignancy

c. How do you investigate for it besides FBC?


▪ Peripheral blood film
▪ Serum Iron and iron binding capacity
▪ Serum ferritin
▪ Serum soluble transferrin receptors
▪ Bone marrow
▪ Other
• Determined by history i.e.
o GIT causes, => endoscopy

2. There was case of a child with pyloric stenosis


a. What feature do you find upon abdominal examination?
i. Visible peristalsis
ii. Positive succussion splash
b. Mention 3 associated electrolyte imbalances due to the condition
i. Hypokalemia
ii. Hypochloremia
iii. Metabolic alkalosis
c. What is the definitive investigation used to diagnose it?

80 | P a g e ...TROY MADAH KAMPIRA...


Abdominal Ultra-sound scan
d. List the Steps to be taken to manage the condition
i. NPO
ii. Fluid resuscitation
iii. Electrolyte balance
iv. Catheterization for monitoring fluid losses
v. Preparing for surgery
vi. Pyloromyotomy

3. There was a case of Transient Tachypnea of the Newborn.


A. What are causes of TTN in newborns?
i. It occurs due to delay in lung fluid clearance that leads to
ineffective gas exchange and results in respiratory distress
ii. Neonatal risk factors: prematurity, Caesarian section without
labour, perinatal asphyxia and maternal gestational diabetes

B. Mention developmental milestones achieved at 3 months


iii. Gross motor: No head lag
iv. Vision: Following moving objects
v. Hearing, language and speech: Vocalizes alone or when
spoken to
vi. Social: smiles responsively

4. There case of TB meningitis


a. Mention drugs to be used to treat the condition
i. Rifampicin (R)
ii. Isoniazid (H)
iii. Ethambutol (E)
iv. Pyrazinamide (Z)
v. Streptomycin (S)

81 | P a g e ...TROY MADAH KAMPIRA...


b. What are the side effects of the mentioned drugs?

Rifampicin
• Toxicity is increased by biliary and hepatic insufficiency.
• On intermittent treatment, six toxicity syndromes have been
recognized; influenzal, abdominal and respiratory symptoms,
shock, renal failure, and thrombocytopenic purpura in 20-30%
patients.

Isoniazid
• Hypersensitive reactions such as rashes may occur.
• Neurotoxicity
• Hepatotoxicity

Ethambutol
• In high doses can get loss of visual acuity and colour vision and
rashes, pruritus, nausea, abdominal pain, confusion, peripheral
neuropathy.
• Should be avoided in very young children
• Ethambutol is contraindicated in renal insufficiency as may lead
to toxicity

Pyrazinamide
• Nausea, upset stomach, vomiting, loss of appetite, mild muscle
or joint pain and fatigue

Streptomycin
• Nausea, vomiting, stomach upset, loss of appetite, spinning
sensation (vertigo), injection site infections (pain, irritation and
redness), Tingling or prickling sensation in the face and rash

5. Explain side effects one can meet in the Extended Programme of


immunisation in Malawi (10 marks)
• Rotavirus vaccine precipitates intussusception
82 | P a g e ...TROY MADAH KAMPIRA...
• Measles vaccine promotes fever and mild rash
• Hepatitis B vaccine may lead to fever and swollen of injection site
• Pentavalent vaccine may lead to swelling of injection site, fever
and anaphylaxis
• Polio vaccine sometimes may lead to paralysis or poliomyelitis
• BCG may sometimes lead to TB and it is contraindicated in
immunosuppressive drugs such as corticosteroids and in
Malignancies and in symptomatic HIV

6. There was a case of DKA, high sugars, vomiting etc.


a. Explain 4 ways in which hypokalemia can develop in the management
of DKA
• Vomiting
• Administration of insulin
• Metabolic alkalosis
• Stimulation of aldosterone

b. Explain ECG changes that rise due to hyperkalemia


vii. Peaking of the T waves.
viii. Increased P-R interval
ix. Flattening of the P wave
x. Widening of the QRS complex occur
xi. Ventricular fibrillation and asystole

c) Explain how you can manage hyperkalemia

7. There was a case: You are in a cardiac clinic and a patient comes cyanotic,
in shock and vomiting
What are your differentials?

83 | P a g e ...TROY MADAH KAMPIRA...


Cyanotic congenital heart diseases
i. Tetralogy of Fallot
ii. Critical pulmonary stenosis
iii. Transposition of great arteries
iv. Truncus arteriosus

a. (If it is TOF) Explain the pathophysiology of hyper cyanotic spells (4


marks)
There is right ventricular outflow obstruction in TOF that tends to get
worse over time and acute episodes of pulmonary infundibular
spasms occur that lead to hyper-cyanotic spells

b. X-ray done showed a narrow mediastinum, explain the narrow


mediastinum
Tetralogy of Fallot. The mediastinum is narrow due to hypoplasia of
the pulmonary valve. The left ventricular apex is displaced laterally and
upward due to right ventricular hypertrophy. The lungs appear anemic
due to reduced pulmonary blood flow secondary to severe
pulmonary stenosis and right to left shunting at the ventricular septal
defect.

c. Explain two other x-ray findings in TOF


v. Small heart (mediastinum)
vi. Boot shaped (uptilted apex)
vii. Pulmonary artery bay + oligemic lung fields
viii. Right sided aortic arch

d. How do you differentiate cyanosis due to cardiac or resp disorder?

84 | P a g e ...TROY MADAH KAMPIRA...


Parameter Cardiac cyanosis Respiratory cyanosis

Respiration Relatively comfortable at Tachypnea, Distress, Retraction


rest
Crying or Efforts Cyanosis may worsen Cyanosis may improve

Chest auscultation Cardiac murmur Rale, Crackle, Wheezing

Chest radiography
Cardiac silhouette Abnormal position or Normal, obliterated cardiac
shape, cardiomegaly margin

Lung fields Normal, Decreased Ground-glass appearance,


vascularity, pulmonary pneumonia, atelectasis,
vascular congestion pneumothorax

EKG Abnormal rhythm or axis Normal

pCO2 Normal or low Usually increased

Response to 100% No or not profound Usually profound

4. There was a case of: malaria parasitemia +++++, fever etc.


a) What is your diagnosis?
Malaria
b) How can you manage the condition above?
Anti-malarial; artesunate IV/IM/rectal 12 hourly or Quinine IV/IM 8hourly
c) later the BCS dropped to 2/5. What is your diagnosis?
Cerebral Malaria

85 | P a g e ...TROY MADAH KAMPIRA...


d) Describe the possible fundoscopic findings of the condition above?
i. Retinal hemorrhages
ii. Retinal whitening
iii. Optical disk oedema
iv. Vascular discolouration
e) Name the common bacteria that will grow in culture with above diagnosis?
Non-typhoidal Salmonella

2014 END OF YEAR PAEDIATRICS QTNs


QUESTION 1
A five-year-old girl is admitted to special care ward with fever, vomiting and
lethargy. On examination she has a temperature of 38.5, a BCS of 3/5; she looks
pale, has a mild jaundice and has tachypnoea and tachycardia. Chest,
cardiovascular and abdominal examination is otherwise normal. She has a blood
glucose of 4mmol/l, MP’s 4+, and PCV of 10%.
a. What is the diagnosis? (2 marks)
Severe Malaria
b. How would you treat the condition in ‘a’ at Queen Elizabeth Central Hospital?
(2 marks)
i. Artesunate (intravenous, intramuscular or suppositories) 12 hourly.
Or Quinine (if artesunate is not available; intravenous or intramuscular) 8 hourly.
ii. Blood transfusion to treat the severe anaemia

c. Two days later, despite appropriate treatment, the patient looks much more
lethargic and her temperature is above 40 degrees Celsius. She is still
86 | P a g e ...TROY MADAH KAMPIRA...
tachypnoeic, has cool peripherals and has capillary refill time of 5 seconds. The
Registrar now suspects that the patient has developed septicaemia. What is the
most likely causative organism for the sepsis in view of the information so far
given? (1)
Streptococcus pneumonia
d. If the patient has Meningitis from the same organism as above, what would be
the positive result on Gram staining of the CSF? (2)
The Gram-positive cocci
The patient’s condition progressively deteriorates. On further examination she
withdraws on painful stimulation, opens eyes but does not follow objects and she
cries inappropriately when stimulated. She has laboured breathing and posturing
movements.
e. What is her Blantyre Coma Score? (1)
Motor response of 1/2, Verbal response of 1/2 & Visual response of 0/1
Total BCS is 2/5
The ophthalmologist is called in to do fundoscopy for features of cerebral
malaria.
f. List two features of malaria retinopathy (2)
i. Retinal haemorrhages
ii. Retinal whitening
iii. Optical disc oedema
iv. Vascular discolouration

QUESTION TWO
a. How would you recognize septic inflammatory response syndrome – five ways

87 | P a g e ...TROY MADAH KAMPIRA...


b. What is the difference between sepsis and SIRS?
c. What are the two common causes of sepsis in Malawi?
i. Streptococcus Pneumonia group B
ii. Staphylococcus
iii. E. coli
d. Outline three things you would do to manage the patient with sepsis
i. Supportive management
• Ensure warmth
• Gentle stimulation if apnoeic
• Respiratory support with oxygen or CPAP if there is severe respiratory
distress or oxygen saturations less than 90%
• If hypoglycaemic, infuse 2ml/kg of 10% dextrose stat
• Give vitamin K if not received already
ii. Definitive management
• Do septic screening which involves lumbar puncture, blood culture, urine
culture and Full blood count which is later followed by IV antibiotics
which are started as soon as possible which include Penicillin or
Cloxacillin and Gentamicin

QUESTION THREE
baby boy is brought in for a six-month review. He was born vaginally at term
from a diabetic mother, with APGAR scores of 2/10 and 5/10 and a birth weight
of 4500g and presentation is said to have been hand prolapse (Hand coming out
first). After the initial resuscitation in labour ward, the baby had been
immediately transferred to Chatinkha nursery where she stayed for a week on
treatment before discharge.

88 | P a g e ...TROY MADAH KAMPIRA...


You are asked to make a problem list for this baby during the perinatal period.
a. List three issues that would be on your problem list (3)

i. Fracture
ii. Nerve injuries
ii. Soft tissue injuries

You are further informed that the baby now has occasional convulsions. The
mother explains that during the seizure, the baby has twitching of the mouth and
fingers on the right side for about 1 minute with drooling from the mouth.
However, the baby remains alert during the episodes.
b. What type of seizure does this baby have (1)

Simple partial seizure


c. If you had seen this baby in Chatinkha nursery soon after birth, what simple
crucial investigation would you have done? (1)

Random Blood Sugar


The mother further explains that now the baby has difficulties in feeding. He is
able to receive feeds by mouth but has explosive coughs with choking as soon as
he attempts to swallow. As a result, he only takes in very small amounts at a
time. You think the child has a poor gag reflex.
d. Which cranial nerves are involved? (2)

i. Glossopharyngeal nerve
ii. Vagus nerve

As part of examination, you find the following on anthropometry: weight for


height 60%; height for age 90%.
e. Interpret these findings (2)

89 | P a g e ...TROY MADAH KAMPIRA...


i. Severe wasting
ii. Normal height

You are asked to examine the right hand. You find that the arm is held in
extension, slightly adducted and internally rotated. The wrist is in flexion and the
forearm in pronation.
f. In view of the signs, what is the diagnosis? (1)

Erb’s Palsy

2013 END OF YEAR PAEDS QTNS

(Same questions as in 2019 End of Paediatrics Rotation SAQs above)

2013 END OF YEAR PAEDIATRICS SAQs

(Same questions as in 2019 End of Paediatrics Rotation SAQs above)

2012 END OF PAEDIATRICS ROTATION (MCQs & SAQs)


1. Which of the following is a feature of WHO stage 3 HIV infection in children?
A Cryptococcal meningitis
B. Kaposi’s sarcoma
C Molluscum contagiosum
D Oral thrush
E Severe unexplained wasting

90 | P a g e ...TROY MADAH KAMPIRA...


2. A four-year-old boy presents with a week’s history of cough and shortness of
breath. A chest x-ray done in on admission shows a dense opacity on the right
side and tracheal deviation to the same side. The most likely diagnosis is:
A Consolidation on the left side
B Lung collapse on the right side
C Pleural effusion on the right side
D Pneumothorax on the right side
E Pneumothorax on the left side

3. A female neonate is born at 28 weeks gestation, via a Caesarean section due to


the mother’s Eclampsia. She is subsequently admitted into Chatinkha Nursery.
There is no other significant antenatal history. On examination she is on oxygen,
breathing at a rate of 80/minute and she has grunting and deep subcostal and
intercostal recessions. She is however pink, has normal pulses and heart sounds. A
chest x-ray shows a diffuse ground glass appearance and air bronchograms. The
most likely diagnosis is:
A Congenital pneumonia
B Meconium aspiration
C Persistent ductus arteriosus
D Respiratory distress syndrome
E Transient tachypnoea of the new-born (TTN)
4. A term baby boy presents to Chatinkha nursery on day three of life with
seizures and poor feeding. He was born through normal vaginal delivery in
hospital with from an HIV negative mother who had presented earlier with
draining liquor. Apgar scores were 9/10 and 10/10 and birth weight was

91 | P a g e ...TROY MADAH KAMPIRA...


3000g. On examination he is irritable, has increased tone and a bulging
fontanelle. The most likely diagnosis is:
a. Congenital hydrocephalus
b. Hypoglycaemia due to poor feeding
c. Hypoxic ischaemic encephalopathy
d. Intraventricular bleed secondary to birth trauma
e. Neonatal meningitis

5. The narrowest part of the upper airway in a child is

A. The carina
B. The terminal bronchiole
C. The right main bronchus
D. The hypopharynx
E. The cricoid ring

6. A 16-month-old child with Down syndrome (trisomy 21) is seen in the


Accident and Emergency Department. Which congenital heart defect is
mostly associated with this syndrome?

a. Complete atrioventricular canal defect


b. Eisenmenger complex
c. Secundum atrial septal defect
d. Transposition of the great arteries
e. Truncus arteriosus

7. A 3-year-old child presents with easy fatigability and often squat for relief
of hypoxic episodes. On examination he is found to have failure to thrive
and a chest X-ray shows a “boot-shaped” heart. Which of the following is
the most likely diagnosis is?
92 | P a g e ...TROY MADAH KAMPIRA...
A. Atrial septal defect
B. Persistent truncus arteriosus
C. Tetralogy of Fallot
D. Transposition of great vessels
E. Ventricular septal defect

8. A female child has just begun to sit on his own, cries when separated from
his parents and laughs often. Developmentally, how old is he?

A. 12 months
B. 14months
C. 4 months
D. 6 months
E. 6 weeks

9. Grace is a 4-year-old who attends A & E following profuse watery


diarrhoea for 5 days and vomiting for 2 days. Her clinical findings are of
lethargy, heart rate of 150 beats per minute, respiratory rate of 35/minute,
capillary refill of 4 seconds and cold peripheries. What is the diagnosis?

A. Gastroenteritis with some dehydration


B. Gastroenteritis with moderate dehydration.
C. Gastroenteritis with hypovolaemic shock
D. Metabolic acidosis
E. Metabolic alkalosis

10.A 13-year-old girl, HIV reactive and not on antiretrovirals, has experienced
burning pains on her left side of the chest since a few days ago. This
morning she noted vesicular eruptions extending laterally almost in a linear

93 | P a g e ...TROY MADAH KAMPIRA...


fashion from left sternal edge. The most likely diagnosis according to this
description is:

a. Chicken pox
b. Cold sores
c. Condyloma acuminata
d. Herpes zoster
e. Kaposi's varicelliform eruption

11. Which of the following is the best definition of severe wasting in a child?

A Height for age SD score <-3


B Height for age <85%
C Weight for height <70%
D Weight for age -3≤SD Score<-2 (70-79%)
E Weight for height -3≤SD Score < -2 (80-89%)

12.A typical neurological finding in a new-born baby with trisomy 21 is:

A Brisk reflexes
B Bulging fontanelle
C Hypertonia
D Hypotonia
E Seizures

13.The most common cause of hypertension in children is

A Endocrine pathologies

94 | P a g e ...TROY MADAH KAMPIRA...


B Intracranial hypertension
C Neurovascular pathology
D Primary cardiac pathology
E Renal pathology

Questions 31, 32, 33: Short Answer questions


Read the questions carefully and write your answers in the spaces provided
14.A 4-year-old boy presents with shortness of breath, cough and fever. On
examination temperature = 39oC, respiratory rate is raised and there is
inter costal recession. Chest x-ray confirms lobar consolidation in the right
lower zone.

a. List 4 other clinical signs that may be apparent in the right lung field:
(4 marks)

i. Reduced air entry

ii. Dullness to percussion


iii. Bronchial breathing
iv. Crackles
v. Increased vocal fremitus or resonance

b. Explain the physiological basis for inter costal recession in this child
(2 marks)

i. Increased negative intrapleural / intrathoracic pressure

ii. Compliant chest wall in young children which deforms easily


95 | P a g e ...TROY MADAH KAMPIRA...
c. Which organism is most likely responsible for this illness (1 mark)

Streptococcus pneumoniae
d. Name an antibiotic or antibiotics that should be started (1 mark)

i. Penicillin and Gentamycin

ii. Ceftriaxone
e. Name 2 other treatments that should be considered (2 marks)

i. Oxygen

ii. IV fluids
iii. NG fluids
iv. Paracetamol

(b) A ten-year-old boy with Sickle Cell disease is described as having


upper motor neurone signs following a stroke affecting the right side of his
body. He has a right sided facial weakness.

f. Which side would you expect the mouth to be deviated to?


(1 mark)
Left side

g. List three parameters that you could test to determine if the optic
nerve (cranial nerve number 2) is involved. (3 marks)

i. Visual acuity
ii. Papillary responses
96 | P a g e ...TROY MADAH KAMPIRA...
iii. visual fields

h. On further examination, he sounds a little bit hoarse and has a soft


stridor. He is also not able to clench his teeth and waggle his jaw
from side to side against resistance.
List the two cranial nerves involved (2 marks)

i. Cranial nerve 5 (Trigeminal)


ii. Cranial nerve 10 (Vagus/recurrent laryngeal)

i. Which cranial nerves are involved in eye movements? (3 marks)


i. Oculomotor (Cranial nerve 3)
ii. Trochlear (Cranial nerve 4)
iii. Abducens nerve (Cranial nerve 6)

j. Finally, you examine the lower limbs. What would the plantar
reflexes be like in this boy’s left leg? (1 mark)
Normal /down going/ Babinski’s negative

2012 END OF YEAR PAEDIATRICS QTNs

1. A baby boy is born at 37 weeks gestation in good condition by spontaneous


vertex delivery following an uneventful pregnancy. He required no
resuscitation at birth and had Apgar of 8 and 10 (at 1 and 5 minutes
respectively). His birth weight was 1650g. How would you classify his
gestation and birth weight?
A. He is not premature and does not have a low birth weight
B. He is not premature, but has a low birth weight
C. He is premature and has a low birth weight
D. He is premature and has a very low birth weight
E. He is premature and does not have a low birth weight

97 | P a g e ...TROY MADAH KAMPIRA...


2. A 5day old preterm infant presents with jaundice, lethargy, and seizures.
Bilirubin level is 25mg/dL (450mmol/L). Which of the following is the most
likely diagnosis?

A. Biliary stenosis
B. Cardiac failure
C. Intussusceptions
D. Kernicterus
E. Respiratory distress syndrome

3. A 10-year-old girl with sickle cell disease presents to hospital with anaemia and
heart failure. Upon investigations she is diagnosed with aplastic crisis.

What is the most likely associated pathogen?

A. Cytomegalovirus
B. Malaria
C. Non-typhoidal salmonella
D. Parvo virus
E. Pneumococcal infection

4. An 8-year-old boy with nephrotic syndrome on long term steroids, presents


with vomiting and painful abdominal distension. He is febrile, well perfused,
has a normal blood pressure and is passing normal urine and stool.

What is the most likely diagnosis?


A. Appendicitis
B. Lobar pneumonia
C. Pancreatitis
D. Pyelonephritis
E. Spontaneous bacterial peritonitis

98 | P a g e ...TROY MADAH KAMPIRA...


5. A four-year-old HIV uninfected boy presents with seizures and headache. He
has a Blantyre coma score of 4/5 on presentation. A lumbar puncture is taken
and the CSF is cloudy. The CSF subsequently grows Streptococcus Pneumoniae.

Which of the following long-term complications is most likely to occur?


A. Cerebral palsy
B. Epilepsy
C. Hearing impairment
D. Hydrocephalus
E. Visual impairment

6. In June 2012, a mother brings her baby at 18 weeks of age for immunization.
She has so far adhered to the WHO immunization guideline.

Which immunization is this baby expecting to receive?

A. DTwP/Hib/HepB; Polio 3; PCV


B. DTwP/Hib/HepB; Polio 2; PCV; Rotavirus
C. DTwP/Hib/HepB; Polio 3; PCV; Rotavirus
D. Measles
E. Rotavirus

7. Grace is a 4-year-old who attends A & E following profuse watery diarrhoea


for 5 days and vomiting for 2 days. Her clinical findings are of lethargy, heart
rate of 150 beats per minute, respiratory rate of 35/minute, capillary refill of 4
seconds and cold peripheries. What is the diagnosis?

A. Cholera
B. Hyponatraemic dehydration
C. Hypovolaemic shock
D. Metabolic acidosis

99 | P a g e ...TROY MADAH KAMPIRA...


E. Severe dehydration secondary to gastroenteritis

21. A two-week-old presents with fever and jaundice. On examination she is


alert, jaundiced and has snuffles, a distended abdomen with hepatomegaly of
4cm. Further examination reveals crumpled skin with peeling in the hands.
The most likely diagnosis is
A. Biliary atresia
B. Candida septicaemia
C. Congenital syphilis
D. Epstein Barr Virus infection
E. Hepatitis B infection

22. A five-year-old boy is diagnosed with minimal change nephritic syndrome.


Urine and blood tests for this patient would show:
A Hypercholesterolemia, hypalbuminaemia, proteinuria
B Hypertension, haematuria, periorbital oedema
C Oliguria, hypocholesterolaemia, proteinuria
D Periorbital oedema, proteinuria, hypertension
E Proteinuria, haematuria, hypercholesterolaemia

23. A Mantoux test done three days before on a 3-year-old immunosuppressed


girl is due for reading. When would the Mantoux be considered positive in this
patient?

a. Hyperaemia diameter 5mm and above


b. Hyperaemia diameter 10mm and above
c. Induration diameter 10 mm and above
d. Induration 5mm and above
e. Induration 5 mm or less

100 | P a g e ...TROY MADAH KAMPIRA...


24. A child received all the correct immunisations according to the current
Malawi immunisation schedule up until he was 6 months of age. He is now one
year old and has not received any vaccines since he was six months of age.
Which of the following immunisations is most likely to have missed?
A. BCG
B. Diphtheria, Tetanus and Pertussis
C. Hepatitis B
D. Measles
E. Polio

26. A 2-year-old boy present with a two-day history of cough and runny nose.
The cough is barking in nature and he has developed noisy breathing. He was
previously well. He was born at term and there were no perinatal problems. He
has never been admitted to hospital and is up to date with all his immunizations.
He is an only child and there is no significant family history. On examination he is
crying and runs off when you try and examine him. He has inspiratory stridor.
He has no cyanosis. He has a respiratory rate of 40. He has tracheal tug and
subcostal recession. What is the most likely cause of this presentation?
A. Adenovirus
B. Haemophilus influenzae type b
C. Parainfluenza virus
D. Respiratory syncytial virus
E. Staphylococcus aureus

27. A 2-year-old girl presents to the paediatric accident and emergency


department with pallor, oedema and not passing urine. She has recently been
given some metronidazole for a bout of bloody diarrhoea. On examination she is
pale, hypertensive and has bilateral ankle oedema.
Which of the following infections is most likely to have resulted in this
presentation?
101 | P a g e ...TROY MADAH KAMPIRA...
A. Giardia
B. Rotavirus
C. Shigella
D. Streptococcus Pneumonia
E. Vibrio Cholerae

28. A 10-year-old girl has a right sided ptosis; a dilated right pupil and her right
eye is deviated inferio-laterally. An abnormality of which of the following cranial
nerves is the most likely to explain these findings?
A. Abducens nerve
B. Oculomotor nerve
C. Optic nerve
D. Trigeminal nerve
E. Trochlear nerve

29. A one day old baby born by at term by spontaneous vertex delivery
weighing 2.8kg presents with poor feeding and ‘sleeping too much’. On
examination temperature is 35.8°C, other vital signs are normal. He is lethargic
and has a high-pitched cry when examined. He has no neck stiffness.
What is the most likely organism causing this presentation?
A. Group B Streptococcus
B. Haemophilus influenzae type B
C. Plasmodium Falciparum
D. Streptococcus Pneumoniae
E. Treponema Pallidum

30. A ten-year-old child has been diagnosed with a left sided consolidation.
What findings on physical examination would be most consistent with this
diagnosis?

102 | P a g e ...TROY MADAH KAMPIRA...


A. Reduced chest expansion on left. Stony dull percussion note on the left,
resonant on the right. Absent breath sounds on the left with decreased
vocal resonance. Normal breath sounds on the right.
B. Reduced chest expansion on the left. Hyper resonant percussion note on
the left, resonant on the right. Decreased breath sounds on the left with
decreased vocal resonance. Normal breath sounds on the right.
C. Reduced chest expansion on the left. Dull percussion note on the left,
resonant on the right. Bronchial breath sounds on the left with increased
vocal resonance. Normal breath sounds on the right.
D. Reduced chest expansion on the right. Stony dull percussion note on the
right, resonant on the left. Absent breath sounds on the right with
decreased vocal resonance. Normal breath sounds on left.
E. Reduced chest expansion on the right. Dull percussion note on the right,
resonant on the left. Bronchial breath sounds on the right with increased
vocal resonance. Normal breath sounds on the left.

(28, 29, 30: Extended matching)


a. Aortic regurgitation

b. Aortic stenosis

c. Coarctation of the aorta

d. Hypoplastic left heart syndrome

e. Mitral regurgitation

f. Patent ductus arteriosus

g. Pulmonary stenosis

h. Tetralogy of Fallot

103 | P a g e ...TROY MADAH KAMPIRA...


i. Tricuspid stenosis

j. Ventricular septal defect

28. A premature new born baby presents on the second day of birth with
increasingly fast breathing. On examination he is tachypnoeic, tachycardic and
hypotensive. she has a hyperactive precordium, bounding peripheral pulses and a
continuous systolic murmur heard best in the left second intercostals space.
Patent Ductus Arteriosus

29. You examine a four-day-old full-term baby boy. You find he has more easily
palpable pulses in the arms than in the groin and has a grade 3/6 systolic murmur
heard over the precordium and the interscapular space.

Coarctation of Aorta

30. A five-year-old boy who has been admitted several times for a heart problem
presents to clinic. On examination he is clubbed, has a tinge of cyanosis, and has a
harsh 4/6 ejection systolic murmur heard best in the left upper sternal border with a
single S2.
Pulmonary stenosis

SHORT ANSWER QUESTIONS


32. A seven-year-old boy presents with recurrent history of cough and shortness
of breath for a period of three months. He reports sweating profusely at night
and easily getting tired. In the few days preceding his admission, his parents had
been very worried about his fast breathing and therefore decided to bring him to
hospital. He recently tested HIV positive but has not yet started on ARV’s. On
examination he is alert and pink. He has diffuse umbilicated papules on his
forehead and cheeks. He is in obvious respiratory distress with nasal flaring and
mild sub costal indrawing. His temperature is 37.5; his heart rate is 160 and is
breathing at 40 cycles per minute.
104 | P a g e ...TROY MADAH KAMPIRA...
a. Basing on these findings, what WHO HIV stage is this patient? (1)

HIV Stage 2
If this patient had a symptomatic pericardial effusion; what would be the
expected findings;
b. On inspection? (2)
i. Increased breathlessness
ii. Raised JVP during inspiration - Kusmall breathing

c. On palpation? (4)

i. Impalpable or barely palpable apex beat


ii. Apex beat not displaced

d. On auscultation? (1)

i. Pericardial rub
ii. Muffled heart sounds

e. How would you confirm your diagnosis of a pericardial effusion? (1)

Echocardiogram
f. What would be the most likely cause of the pericardial effusion in this
patient? (1)

Tuberculosis

2012 DEFERRED EXAM - END OF YEAR PAEDIATRICS QTNs


Same as 2018 End of year Supplementary exam above

105 | P a g e ...TROY MADAH KAMPIRA...


2011 END OF YEAR PAEDIATRICS MCQs & SAQs

7. Which of the following is consistent with stage four HIV according to the
WHO clinical staging criteria?

A. Pulmonary tuberculosis
B. Lymphoid interstitial pneumonitis
C. Severe recurrent bacterial pneumonia (2 episodes in 6 months)
D. Pneumocystis Jiroveci Pneumonia
E. HIV associated chronic lung disease
Answer D

8. Which of the following respiratory rates would you consider normal for a
healthy two-year-old boy?

A. 12
B. 18
C. 33
D. 41
E. 62
Answer C

9. Which of the following is a feature of vesicular breath sounds?

A. Breath sounds steadily increase in intensity through inspiration.


B. The expiratory phase is louder than the inspiratory phase
C. There is a pause between the inspiratory and expiratory phase
D. Vocal resonance is increased.
E. Whispering pectoriloquy is usually heard
Answer A

10. A 3-month-old baby girl presents with a one-day history of cough, fever and
fast breathing. Which one of the following would lead her to being classified as
very severe pneumonia according to the WHO classification?

106 | P a g e ...TROY MADAH KAMPIRA...


A. Respiratory rate 61
B. Crackles on auscultation
C. Vomiting all feeds
D. Nasal flaring
E. Subcostal recession
Answer C

16. An eleven-year-old girl presents to the pediatric emergency department with a


two-months history of cough and shortness of breath on exertion. On
examination there is no finger clubbing and no cyanosis. She has intercostal and
subcostal recessions and a respiratory rate of 40 bpm. Her apex beat is visible and
is palpated in the seventh left intercostal space, anterior axillary line. She has a
normal first and second heart sound with a grade 3/6 pansystolic murmur heard
loudest at the apex, which radiates to the axilla.

What is the most likely explanation for these physical findings?

A. Ventriculo-septal defect
B. Mitral regurgitation
C. Dilated cardiomyopathy
D. Tetralogy of Fallot
E. Patent Ductus Arteriosus
Answer B

21. A child attends the pediatric A&E department and is clerked by the intern who
diagnoses a “simple febrile convulsion”. Which of the following would be most in
keeping with the diagnosis of a simple febrile convulsion?

A. A seizure involving jerking of the right arm only


B. A seizure which stops spontaneously after 20 minutes
C. Three seizures occurring within 2 hours
D. A brief generalized seizure with urinary incontinence
107 | P a g e ...TROY MADAH KAMPIRA...
E. A seizure in a 4-month-old child
Answer D
26. A Term baby is born by Caesarian section with a birth weight of 3.5kg. At
one minute of age, he is blue, very floppy, has a heart rate of 140 bpm, he has
irregular breathing and a feeble cry when stimulated.

What is his Apgar score at one minute?

A. 1/10
B. 2/10
C. 3/10
D. 4/10
E. 6/10
Answer D

27. A vigorous baby boy is admitted to Chatinkha nursery weighing 1.2kg


following a spontaneous vertex delivery. Which of the following features would
be most consistent with the baby being preterm rather than full term with
intrauterine growth restriction?

A. The baby has leathery, cracked and wrinkled skin


B. The baby lies with a flexed posture
C. The baby’s ear has a firm pinna, which recoils quickly
D. The baby has a 5mm breast bud
E. The baby has abundant lanugo hair over his back and shoulders
Answer E

29. A term baby, weighing 3000g is admitted to Chatinkha nursery following a


Caesarian section for delayed second stage of labor. Following delivery, he is
resuscitated with bag and mask ventilation before onset of regular respirations at
6 minutes. The Apgar scores were 1 at one minute, 4 at 5 minutes and 8 at 10
minutes. Initially he is lethargic and floppy with few spontaneous movements. He
has three short lived seizures and is given a loading dose of phenobarbitone.
After five days he is sucking well and has had no further seizures. He has normal
108 | P a g e ...TROY MADAH KAMPIRA...
tone and symmetrical movements. He is discharged home. Which of the
following would most accurately describe his diagnosis?

A. Mild birth asphyxia


B. Hypoxic ischemic encephalopathy stage 2
C. Hypoxic ischemic encephalopathy stage 3
D. Low Apgar score
E. Benign neonatal seizures
Answer B

32. a) List six features found in malaria that would lead to the malaria being
classified as severe. (6 marks)

Cerebral Malaria
Severe Anemia
Renal Failure
Pulmonary oedema/acute respiratory distress syndrome
Hypoglycemia
Circulatory collapse/Shock
Abnormal bleeding/DIC
Repeated generalized convulsions
Acidemia/Acidosis
Hemoglobinuria
Impaired consciousness
Prostration/weakness
Hyperparasitemia
Hyperpyrexia
Hyperbilirubinemia/jaundice

b) List six different complications of bacterial meningitis. Three should be


early complications and three should be late complications.

a. Early complications (3 marks)

109 | P a g e ...TROY MADAH KAMPIRA...


Seizures
Hypoglycemia
Sepsis
Raised intracranial pressure
Cerebral oedema
Vasculitis/Thrombosis
Focal neurological deficits
Subdural effusion, empyema
Hydrocephalus
Syndrome of inappropriate ADH secretion, hyponatremia
Cranial nerve palsy
Brain abscess

b. Late complications (3 marks)


Focal neurological deficits
Subdural effusion, empyema
Hydrocephalus
Cranial nerve palsy
Epilepsy
Cerebral palsy
Developmental delay
Hearing impairment
Visual impairment

33. a) List five clinical signs of kwashiorkor (5 marks)

Oedema
Sparse, thin, brittle, depigmented hair
Unhappy, miserable, apathetic, irritable
Angular stomatitis
Smooth tongue
Flaky paint dermatitis
Abdominal distension
Low weight
110 | P a g e ...TROY MADAH KAMPIRA...
Low height
Muscle wasting
Enlarged liver
Pallor
Parkinsonian shakes/Kahn’s syndrome
Purpura (uncommon)
Small tonsils
Hypothermia
Jaundice
Xeropthalmia, corneal ulceration

2011 DEFERRED EXAM END OF YEAR MCQS

2. Which of the following breath sounds are usually heard loudest during
expiration?

A. Crackles
B. Wheeze.............
C. Bronchial breath sounds
D. Vesicular breath sounds
E. Stridor

7. Regarding children on ART one of the following is right

A Skin reactions to drugs are always life threatening


B New active TB definitely indicates treatment failure
C IRIS is not a feature if immunity is improving on treatment
D *Improvement in growth parameters is a good indicator of response to
treatment
E Stage 1 conditions cannot be started on treatment
8. Herpes virus 6 is associated with

111 | P a g e ...TROY MADAH KAMPIRA...


A Erythema toxicum
B Eczema herpeticum
C *Roseola infantum
D Erythema marginatum
E Exanthema subitum

9. One of the conditions below is an acyanotic cardiac abnormality


A Renal artery stenosis
B *Coarctation of the aorta
C Pulmonary artery stenosis
D Tetralogy of Fallot
E Transposition of the great arteries

17.The most common pathogen isolated in children with bronchiolitis is


a. Adenovirus
b. Parainfluenza virus
c. Respiratory syncytial virus
d. Rhino virus
e. Herpes simplex virus
ANSWER: C

21.Which of the following statements concerning TB meningitis is true?


a. Is commonest in children over 2 years of age
b. Examination of CSF always show a lymphocytic predominance
c. Requires treatment with four TB drugs for 10mnths
d. May cause an obstructive hydrocephalus
e. Is usually associated with a strong positive Mantoux result

112 | P a g e ...TROY MADAH KAMPIRA...


22. Grace is a 4-year-old who attends A & E following profuse watery
diarrhoea for 5 days and vomiting for 2 days. Her clinical findings are of
lethargy, heart rate of 150 beats per minute, respiratory rate of 35/minute,
capillary refill of 4 seconds and cold peripheries. The following statements
are true.

A. The heart rate is within the acceptable normal range for this child.
B. A capilliary refill is measured by holding a toe or finger for 3 seconds,
letting go and seeing if perfusion returns within 2 seconds.
C. A capilliary refill time of 4 seconds can be normal
D. The above findings suggest that this child is shocked
E. The child has gastroenteritis with moderate dehydration.
ANSWER: D

23. A 3-month-old baby is brought to the Accidents and Emergency unit


with a bulging fontanelle
a. One sign of increased intracranial pressure in babies is irritability
b. This baby is likely to have a positive Kernig’s and Brudzinski’s sign
c. If the baby had retinal haemorrhages, the cause is likely to be
meningitis
d. If a baby will not stop crying, the mother should shake the baby until
the crying stops
e. Cushing’s triad includes tachycardia, hypotension and irregular
respirations
ANSWER: A

24. Acute bronchiolitis is

A. Is usually associated with high fever and rash


B. Usually associated with bilateral infiltrates on chest x-ray
C. Commonly associated with retractions, tachypnoea, and wheezing
113 | P a g e ...TROY MADAH KAMPIRA...
D. Characterized by the absence of cough despite respiratory distress
E. Most common between 2 and 5 years of age
ANSWER: C

DIRECTIONS FOR QUESTION 27-30

Each set of matching questions in this section consists of a list of options followed
by several numbered items. For each numbered item select the ONE lettered
option with which it is most closely associated.

For characteristics of renal disease given below, select the urinary finding most
closely associated.

A. Red blood cell (RBC) casts


B. White blood cell (WBC) casts
C. Hyaline casts
D. Renal tubular casts
E. Waxy casts
F. Fatty casts

25. Urinary finding in an 8-year-old boy with generalized oedema and a


24-hour urine protein greater than 3.5g
ANSWER: F

26. Urinary finding that distinguishes acute pyelonephritis from acute


cystitis
ANSWER: B

27. Urinary finding that would be expected in a 12-year-old child who


develops hypertension, periorbital oedema, and smoky coloured urine 2
weeks after recovering from impetigo.
ANSWER: A

114 | P a g e ...TROY MADAH KAMPIRA...


28. A six-year-old boy with generalized oedema and has recently
developed spontaneous bacterial peritonitis
ANSWER: F

OTHER EXAM PAPERS WITH UNKNOWN YEARS

115 | P a g e ...TROY MADAH KAMPIRA...


116 | P a g e ...TROY MADAH KAMPIRA...
117 | P a g e ...TROY MADAH KAMPIRA...
118 | P a g e ...TROY MADAH KAMPIRA...
119 | P a g e ...TROY MADAH KAMPIRA...
120 | P a g e ...TROY MADAH KAMPIRA...
121 | P a g e ...TROY MADAH KAMPIRA...
122 | P a g e ...TROY MADAH KAMPIRA...
123 | P a g e ...TROY MADAH KAMPIRA...
124 | P a g e ...TROY MADAH KAMPIRA...
125 | P a g e ...TROY MADAH KAMPIRA...
126 | P a g e ...TROY MADAH KAMPIRA...
127 | P a g e ...TROY MADAH KAMPIRA...
128 | P a g e ...TROY MADAH KAMPIRA...
129 | P a g e ...TROY MADAH KAMPIRA...
130 | P a g e ...TROY MADAH KAMPIRA...
131 | P a g e ...TROY MADAH KAMPIRA...
132 | P a g e ...TROY MADAH KAMPIRA...
133 | P a g e ...TROY MADAH KAMPIRA...
134 | P a g e ...TROY MADAH KAMPIRA...
135 | P a g e ...TROY MADAH KAMPIRA...
136 | P a g e ...TROY MADAH KAMPIRA...
137 | P a g e ...TROY MADAH KAMPIRA...
138 | P a g e ...TROY MADAH KAMPIRA...
139 | P a g e ...TROY MADAH KAMPIRA...
140 | P a g e ...TROY MADAH KAMPIRA...
141 | P a g e ...TROY MADAH KAMPIRA...
142 | P a g e ...TROY MADAH KAMPIRA...
143 | P a g e ...TROY MADAH KAMPIRA...
144 | P a g e ...TROY MADAH KAMPIRA...
145 | P a g e ...TROY MADAH KAMPIRA...
146 | P a g e ...TROY MADAH KAMPIRA...
147 | P a g e ...TROY MADAH KAMPIRA...
1993, 1994, 1995. 1996. 1997. 1998, END OF PAEDIATRICS ROTATION SAQs

148 | P a g e ...TROY MADAH KAMPIRA...


149 | P a g e ...TROY MADAH KAMPIRA...
150 | P a g e ...TROY MADAH KAMPIRA...
151 | P a g e ...TROY MADAH KAMPIRA...
152 | P a g e ...TROY MADAH KAMPIRA...
153 | P a g e ...TROY MADAH KAMPIRA...
154 | P a g e ...TROY MADAH KAMPIRA...
155 | P a g e ...TROY MADAH KAMPIRA...
156 | P a g e ...TROY MADAH KAMPIRA...
157 | P a g e ...TROY MADAH KAMPIRA...
158 | P a g e ...TROY MADAH KAMPIRA...
159 | P a g e ...TROY MADAH KAMPIRA...
160 | P a g e ...TROY MADAH KAMPIRA...
161 | P a g e ...TROY MADAH KAMPIRA...
162 | P a g e ...TROY MADAH KAMPIRA...
163 | P a g e ...TROY MADAH KAMPIRA...
164 | P a g e ...TROY MADAH KAMPIRA...
165 | P a g e ...TROY MADAH KAMPIRA...
166 | P a g e ...TROY MADAH KAMPIRA...
167 | P a g e ...TROY MADAH KAMPIRA...
ANOTHER PAEDIATRICS PAPER OF UNKNOWN YEAR
1. A term baby, weighing 3000g is admitted to Chatinkha nursery following a
Caesarian section for delayed second stage of labor. Following delivery, he
is resuscitated with bag and mask ventilation before onset of regular
respirations at 6 minutes. The Apgar scores were 1 at one minute, 4 at 5
minutes and 8 at 10 minutes. Initially he is lethargic and floppy with few
spontaneous movements. He has three short lived seizures and is given a
loading dose of phenobarbitone. After five days he is sucking well and has
had no further seizures. He has normal tone and symmetrical movements.
He is discharged home. Which of the following would most accurately
describe his diagnosis?

a. Mild birth asphyxia


b. Hypoxic ischaemic encephalopathy stage 2
c. Hypoxic ischaemic encephalopathy stage 3
d. Low Apgar score
e. Benign neonatal seizures

2. A child attends the Paediatric A&E department and is clerked by the intern.
The mother describes an episode of ‘twitching’ of the right hand
accompanied by about 5 minutes of unconsciousness and then slow return
to full alertness. This type of seizure would be described as:

a) Absence seizure
b) Generalise tonic-clonic seizure
c) Complex partial seizure
d) Generalised clonic seizure
e) Simple focal seizure

3. A 28-day old floppy male neonate is brought to paediatric Accidents and


Emergency department. How should he be triaged?

168 | P a g e ...TROY MADAH KAMPIRA...


a) As emergency
b) As priority
c) As priority after checking the blood glucose.
d) As queue
e) As queue after giving anticonvulsant.

4. A vigorous baby boy is admitted to Chatinkha nursery weighing 1.8kg


following a spontaneous vaginal delivery. Which of the following features
would suggest that the baby is full term with intrauterine growth restriction
rather than preterm?

a. The baby has no palpable breast bud tissue


b. The testes are palpable in the inguinal canal
c. The baby’s ear has a firm pinna, which recoils quickly
d. The baby has no sucking reflex
e. The baby has abundant lanugo hair over his back and shoulders

SHORT ANSWER QUESTIONS (SAQ) (20 Marks)

1. History:
Sami, a 9-month-old boy, has been referred to the community Paediatric clinic by
his health visitor because his head circumference is on the 99.6th centile. He was
born at 38 weeks’ gestation by spontaneous vaginal delivery, following an
uneventful pregnancy to a Samoan woman. His birth weight was 4.3 kg (91st
centile) and his head circumference at birth was 38 cm (98th centile). This is the
mother’s third baby, but the other two children have a different father. Sami
doesn’t sleep very well, waking up to three times per night, and this was the
main reason his mother went to see her health visitor. Sami is otherwise well and
has not had any previous medical problems. He has been able to sit unsupported
for the last 2 months, he crawls, he can use either hand to pick up raisins or grains
169 | P a g e ...TROY MADAH KAMPIRA...
of rice. His mother has no concerns about his vision or hearing and he passed his
newborn hearing screen. He is still breast-feeding but also eats purées and finger
foods. The mother, her partner and the three children live in a two-bedroom flat.
The growth chart in his parent-held child health record is shown in Figure 63.1

O/E:
Sami is a well-looking child, who smiles and babbles continuously. He is not
dysmorphic. His weight is 10.5 kg (91st centile), his length is 76.2 cm (98th
centile) and his head circumference is 49.7 cm (99.6th centile). Cardiovascular
170 | P a g e ...TROY MADAH KAMPIRA...
and respiratory examinations are normal. His anterior fontanelle is almost closed.
Neurological examination reveals normal tone, power and reflexes in the upper
and lower limbs.

• Mention 4 causes of macrocephaly and for each cause give a key


physical examination finding? (4 marks)

• Mention 4 symptoms of raised intracranial pressure in a 9 months


year old baby? (2 marks)
• Vomiting
• Lethargy
• Irritability
• Poor feeding

• In the patient described above, what further questions would you ask
in the categories listed below? For each question give a reason? (3 marks)

i) Developmental history
ii) Social History
iii) Family History

171 | P a g e ...TROY MADAH KAMPIRA...


• What is the diagnosis of the patient from the facts presented above?
Why? (1 mark)
• The child has normal development and his growth chart shows he has
always had the large head. The child is normal.

ANOTHER EXAM PAPER WITH PAEDIATRICS QTNS

1. a) On a night of Pediatric call, you are just finishing your dinner when your
pager goes off and you are quickly in touch with your senior resident
Bentley. He has just received a new consult from the emergency
department about a 7-year-old girl presenting with acute onset bloody
diarrhea who will likely require admission to the General Pediatric service.
Unfortunately, Bentley is in a rush to deal with another issue on the wards
and doesn’t have time to fill you in with more details. He has confirmed
that the patient is stable, and has asked you to going ahead and start seeing
the patient. He is looking forward to hearing your impression and plan.

a) You walk down towards the Pediatric Emergency department you take the
opportunity to start putting together a differential diagnosis based on the little
information that you do have. List and justify 10 differential diagnoses
(5marks)

b) What four investigations would you like to order? (2marks)

You cross-check in the health passport and you find a sticker with the
following results:

• CBC: Hb 90, Plts 40, WBC, 22


• Electrolytes: normal, other than high-normal K
• Urea: 20
• Creatinine: 180

172 | P a g e ...TROY MADAH KAMPIRA...


c) Which among your list of differentials is most consistent with the results and
the history above? (1mark)

d) What are the key features of your answer in c above? List 4 (2marks)

ANOTHER PAPER WITH PAEDIATRICS QTNS

7. Nephrotic syndrome
(a) Symptoms consistent with nephrotic syndrome
❖ Peri-orbital edema as well as peripheral pitting edema
❖ Abdominal pain and diarrhea
❖ Anorexia

(b) Four lab tests abnormalities seen in nephrotic syndrome


❖ 3+ or 4+ proteinuria
❖ Microscopic hematuria
❖ Serum cholesterol & trigycemia levels are elevated
❖ Serum albumin less than 2.5g/dl

(c) Complications of nephrotic syndrome


❖ Increased risk infections e.g., pyelonephritis, peritonitis due to loss of
complement factor C3b & immunoglobin in urine
❖ Hypercoagulable state due to vascular stasis, increase in hepatic
production of fibrinogen and other clotting factors (stimulated by
hypoalbuminemia)

(d) Investigations in nephrotic syndrome


❖ Urine dipstick
❖ FBC
❖ Urine microscopy and culture
❖ U & Es
173 | P a g e ...TROY MADAH KAMPIRA...
❖ LFTs

8. Causes of stridor in children


❖ Laryngomalacia
❖ Croup
❖ Diphtheria
❖ Inhaled foreign body
❖ Acute epiglottitis

(b) Characteristics that are typical of mitral regurgitation murmur


❖ Best heard at the apex
❖ Blowing pansystolic murmur, radiating from the apex to the axilla
❖ Often associated with an S3 because of the left atrial volume overload

(c) Pathogenesis of rheumatic heart disease (RHD)


❖ RHD is a result of an episode (or episodes) of acute rheumatic fever during
which inflammation of endocardium occurs, chiefly affecting the mitral and
aortic valves. The inflammation settles with scaring, fibrosis, and deformity
of the valve, which may lead to heart failure over time

10. Maintenance of fluid of a 32kg patient


10×100=1000ml
10×50=500ml
12×20=240ml
Therefore, he/she needs (1000+500+240) ml
=1740ml/24hours
=72.5ml/hr
11. (a) DKA management, first thing to do
❖ ABC
❖ Fluids-NS 10ml/kg std, max 30ml/kg
174 | P a g e ...TROY MADAH KAMPIRA...
❖ Monitoring, blood glucose, U & Es
11. Jaundice
(a) Causes of obstructive jaundice
❖ Biliary atresia
❖ Gallstones
❖ Bile duct obstruction by ascaris
❖ Enlarged TB lymph nodes or lymphoma

(b) Clinical presentation of obstructive jaundice


❖ Yellowing of skin and sclera of the eyes
❖ Paler stools
❖ darker urine
❖ Intense itching

13. DM
(a) Pathophysiology of type 1 diabetes
❖ Failure to secrete insulin due to damage to beta-cells caused by viral
infection or autoimmune disease
(b) Pathophysiology of type 2 diabetes
❖ Impaired responsiveness to insulin as a result of decreased formation
or sensitivity of cell surface insulin receptors

(c) Name two other types of diabetes


❖ Gestational diabetes
❖ Diabetes LADA

(d) A patient requires oral drug to treat his type 2 diabetes. Which two oral
antidiabetic drugs are currently recommended in Malawi National Treatment
Guidelines? How do you decide which drug to start the patient on?
❖ Metformin- in obese patients
❖ Glibenclamide- in non-obese patients

175 | P a g e ...TROY MADAH KAMPIRA...


14. Pneumonia
(a) Describe WHO treatment protocol for severe pneumonia
❖ Penicillin
❖ Chloramphenicol
❖ If salmonella is considered, add ciprofloxacin
❖ If no improvement, give erythromycin & gentamycin

(b) Clinical signs and WHO diagnosis of pneumonia


❖ In children under five years, who have cough, and/or difficulty breathing,
with or without fever; pneumonia is diagnosed by the presence of either
fast breathing or lower chest wall indrawing. Wheezing more common in
viral infections

(c) List common organisms causing pneumonia


❖ Streptococcus pneumonia
❖ Haemophilus influenza type b (Hib)
❖ RSV
❖ Pneumocystis jiroveci

15. Five forms of severe malaria


❖ Cerebral malaria
❖ Severe anemia
❖ Child with prostration
❖ Malaria with 2 or more convulsions
❖ Malaria with hypoglycemia (blood glucose less than 2.2 mmol/L)

ANOTHER EXAM PAPER WITH PAEDIATRICS QTNS


1. The most feared adverse outcome of iron deficiency anemia is:
A. Pica

176 | P a g e ...TROY MADAH KAMPIRA...


B. High output heart failure
C. Neurodevelopmental delay
D. Anorexia
E. Irritability

2. It is recommended that babies who are exclusively breastfed be exposed to


adequate sunlight to prevent vitamin D deficiency. This is because: A. Most of
breastfed babies spent a lot of time indoors
B. Most people who breastfeeding do not expose their babies to
sunlight
C. Breastmilk contains less vitamin D
D. Globally, most breastfed live areas with minimal sunshine.
E. Conversion of 7-dehydrochlesterol to vitamin D3 is facilitated by
breastmilk.

3. Children height should be measured on a stadiometer after the age of


A. 1 year
B. 2 years
C. 3 years
D. 4 years
E. 5 years

4. Calculation of the mid-parental height is very important as it gives the


health worker provider an indication of:
A. Consanguinity
B. Genetic diseases in the family
C. Endocrine disorder
D. Genetic height potential
E. Idiopathic short stature

5. The two most important diagnostic information in the assessment of short


stature will come from:
177 | P a g e ...TROY MADAH KAMPIRA...
A. Liver function tests (LFTs) and molecular genetics
B. Growth chart and mid-parental height
C. Karyotyping and gene-mapping
D. Hormonal analysis and brain imaging
E. Bone x-rays (bone age) and developmental assessment

6. Although iron deficiency anemia is common in children with Protein Energy


Malnutrition (PEM), iron is only given during rehabilitation because:
A. Iron interferes with absorption of milk
B. Iron is not absorbed in the stabilization phase
C. Iron growth of pathogens
D. Iron cause abdominal pain in stabilization phase
E. Iron causes nausea and vomiting in stabilization phase

7. Below which measurement is considered to be low-weight-for-age


according to the WHO growth charts?
A. 0-z score
B. +1 z-score
C. +2 z-score
D. -3 z-score
E. -2 z-score

8. Which of the following is a contraindication to live vaccines?


A. HIV infected child
B. A child with severe malnutrition
C. A child on chemotherapy
D. A premature baby
E. A child with a mild upper respiratory tract infection

9. From the information given in the options below, which one is a clear
indication for admission to the Nutritional Rehabilitation Unit (NRU)?

178 | P a g e ...TROY MADAH KAMPIRA...


A. 7 months old child with oedema of +
B. 12 months old child with oedema of ++
C. 3 years old child MUAC <11.5cm
D. Child less than 6 months and a weight-for-height of less than -3 z-
scores
E. E. 10 months old child with a weight-for-height of less than -3 z-scores

Matching questions
Select the single best option from the list of options (A-E) given below for each
stem (1013). Each option may be used more than once. You will receive 1 marks
for a correct answer and 0 for an incorrect answer.
A. First year of life
B. Childhood
C. Adolescent
D. A, B, C
E. Just before puberty

10.During this period linear growth is very rapid

11. During this period, growth is mainly influenced by nutrition

12.During this period a stable psychosocial environment is a prerequisite for


normal growth

13.The slowest rate of growth occurs during this period


Select the single best option from the list of micronutrients (A-D) given below for
each stem (14-16). Each option may be used more than once. You will receive 1
mark for a correct answer and 0 for an incorrect answer.

179 | P a g e ...TROY MADAH KAMPIRA...


A. Vitamin K
B. Vitamin D
C. iodine
D. Vitamin A

14.Deficiency may be induced by use of broad-spectrum antibiotics

15.Its deficiency may cause mental retardation

16.it maintains the integrity of the epithelium in the respiratory and


gastrointestinal tracts

Select the single best option from the list of anthropometric measurements (A-E)
given below for each stem (17-21). Each option may be used more than once.
You will receive 1 mark for a correct answer and 0 for an incorrect answer.
A. Mid-upper arm circumference
B. Weight-for-height
C. Height/length-for -age
D. BMI
E. Head circumference

17.Useful to measure stunting

18.Useful to measure wasting

19.Used in children more than 2 years of age


20. Is the last to slow down with chronic malnutrition especially in
children less than 2 years.

21.Useful for community screening of malnutrition

180 | P a g e ...TROY MADAH KAMPIRA...


Select the single best option from the list of types of Protein Energy Malnutrition
(A-B) given below for each stem (22-27). Each option may be used more than
once. You will receive 1 mark for a correct answer and 0 for an incorrect answer.
A. Marasmus
B. Kwashiorkor

22. Hepatomegaly secondary to fatty infiltration is a feature


23. Flaky paint dermatitis may occur
24. Moon face is characteristic
25. Deficiency of some nutrients, particularly those involved with anti-
oxidant protection
26. Is the childhood equivalent of starvation.
27. Hair changes (depigmented hair, dry, sparse, brittle)

Select the single best option from the list of types of prevention of disease (A-C)
given below for each stem (28-31). Each option may be used more than once.
You will receive 1 mark for a correct answer and 0 for an incorrect answer.
A. Primary prevention
B. Secondary prevention
C. Tertiary prevention

28. Growth monitoring


29. Fortifying food with folic acid
30. Use of oral rehydration therapy in diarrhea
31.Physiotherapy for cerebral palsy

Select the single best option from the list of types of vaccines (A-C) given below
for each stem (32-34). Each option may be used more than once. You will
receive 1 mark for a correct answer and 0 for an incorrect answer.
181 | P a g e ...TROY MADAH KAMPIRA...
A. Toxoid vaccine
B. Contains polysaccharide components
C. Contains live attenuated organisms

32. BCG vaccine


33. Measles vaccine
34. Haemophilus influenzae type b vaccine

Suspected answers for the question paper above


1. B 2. C 3. A 4. D 5. B 6. B 7. E 8. C 9. D

Matching questions answers


10. A 11. A 12. B 13. B 14. A 15. C 16. A 17. C 18. B 19. D
20. E 21. A 22. B 23. B 24. B 25. B 26. A 27. B 28. B 29. A
30. C 31. C 32. C 33. C 34. B

ANOTHER EXAM PAPER WITH PAEDIATRICS QTNS


You are reviewing maternal hospital records of a sick newborn who has been
admitted to the neonatal unit. Match the maternal conditions A – E to the
associated newborn condition 1-5 below. Each option may be used once, more
than once or not at all.
A. Maternal severe oligohydramnios on antenatal ultrasound
B. Maternal polyhydramnios on antenatal ultrasound
C. Mother is Rh Negative and baby is Rh positive
182 | P a g e ...TROY MADAH KAMPIRA...
D. Mother had ruptured membranes for 36 hours before delivery.
E. Mother had toxemia of pregnancy

1. Anencephaly
2. Hydrops fetalis
3. Intrauterine growth retardation (IUGR)
4. Neonatal meningitis
5. Bilateral renal agenesis

6. The preterm infant is predisposed to heat loss because they have a high
ratio of a surface area to body weight. Which one of the following statements
is the best comparison of preterm infants and adults regarding their surface
area to body weight ratios?

A. The surface area to body weight of preterm infants is 2 times more


than that of adults B.
B. The surface area to body weight of preterm infants is 3 times more
than that of adults
C. The surface area to body weight of preterm infants is 4 times more
than that of adults
D. The surface area to body weight of preterm infants is 5 times more
than that of adults
E. The surface area to body weight of preterm infants is 6 times more
than that of adults

7. Which of the following is NOT one of the reasons why preterm infants are
prone to heat loss?
A. They have little insulating subcutaneous fat
B. They have reduced glycogen reserves
C. They have little brown fat
D. They have small heads relative to body size

183 | P a g e ...TROY MADAH KAMPIRA...


E. They have a hypotonic (“frog”) posture

Match the mechanism of heat loss (A-D) in the newborn to the best contributing
factor of each method of head loss in questions 8-11 below. Each option may be
used once, more than once or not at all.
A. Radiation
B. Conduction
C. Convection
D. Evaporation

8. Heat loss by this mechanism mainly occurs when babies skin is wet.

9. Heat loss by this mechanism mainly occurs when there is moving air e.g., if
windows in the labor ward are left open.

10. Heat loss by this mechanism is worse when the linen and baby
mattress on the resuscitaire are not prewarmed.

11. Heat lose occurs by this method to colder nearby objects not in
contact with the baby.

Match the aspects of fetal circulation listed from A- B below to the correct
descriptions in questions 12- 15 below. Each option may be used once, more than
once or not at all.
A. Right ventricular ejection volume
B. Left ventricular ejection volume

12. Mainly supplies the brain


13. Supplies the coronary arteries
184 | P a g e ...TROY MADAH KAMPIRA...
14. Has a high partial pressure of oxygen
15. Mainly supplies the upper limbs

16. Which of the following best explains why newborns have a twofold
to threefold greater rate of bilirubin production compared with adults?
A. Newborns have immature pathways of bilirubin metabolism
B. Increased RBC mass (higher hematocrit) in the newborn
C. Shortened erythrocyte life span of 70-90 days compared with the
120-day erythrocyte life span in adults
D. D. B and C
E. All of the above

Match the type of bilirubin listed in A-B with the best characteristic in question 17-
21. Each option may be used once, more than once or not at all.
A. Conjugated bilirubin
B. Unconjugated bilirubin

17. This type of bilirubin is lipid soluble


18. This type of bilirubin is toxic to the central nervous system
19. This type of bilirubin can be displaced from albumin by drugs such as
sulfisoxazole
20. This type of bilirubin is capable of being excreted in the urine
21. The placenta in impermeable to this type of bilirubin
Match the following causes of prolonged jaundice in the newborn (A-D ) to their
clinical presentations in questions 22 –25. Each option may be used once, more
than once or not at all.
A. Congenital hypothyroidism
B. Galactosemia
185 | P a g e ...TROY MADAH KAMPIRA...
C. Hirschsprung disease
D. Biliary atresia

22. 6 weeks old with congenital cataract


23. A 5 weeks old infant with constipation, hypotonia and hypothermic
episodes
24. A 2 months old baby with pale stools
25. A 3 weeks old baby with a history of delay in passage of meconium
(>24 hours)

Match the causes of isoimmmune hemolytic anemia and jaundice in the newborn
(A- B) to the characteristics in question 26 –29 below. Each option may be used
once, more than once or not at all.
A. Rh incompatibility
B. ABO incompatibility

26. More common in infants with blood type A or B who are born to
mothers with blood type O
27. Incidence can be reduced by administration of anti-D gamma globulin
to at-risk mothers
28. Can cause hemolysis in first pregnancy
29. Usually does not cause hemolysis in first pregnancy

Match the causes of jaundice in newborn (A- C) to their characteristics in


questions 30 – 32 below. Each option may be used once, more than once or not
at all.
A. Syphilis
B. Toxoplasmosis
C. Hepatitis B
186 | P a g e ...TROY MADAH KAMPIRA...
30. Perinatal infection can be prevented by vaccinating the babies within
72 hours if the mother has been found to carry the infection.

31. The affected newborn should be treatment is with intravenous


penicillin for 10-14 days.

32. Affected newborn may have hydrocephaly or microcephaly or


intracranial calcifications,

Match the causes of respiratory distress in newborn babies (options A- E) with


their characteristics 33-43. Each option may be used once, more than once or not
at all.
A. Respiratory distress syndrome
B. Transient tachypnea of the newborn
C. Meconium aspiration syndrome
D. Primary Pulmonary hypertension of the newborn
E. Bacterial pneumonia

33. A risk factor includes precipitous delivery


34. This is predominantly a problem of term and post-term deliveries
35. This condition is usually considered clinically as a sign of fetal distress
36. A chest radiograph may show fluid in the lung fissures
37. This condition can cause pneumothoraxes as a complication
38. Significant right-to-left shunting through a patent foramen ovale or
through PDA is characteristic of this condition.
39. This condition can cause a ball-valve effect, leading to air trapping,
overdistention of the alveoli.
40. Diagnosis is confirmed by echocardiograph examination
41. Chemical pneumonitis is a feature of this condition
187 | P a g e ...TROY MADAH KAMPIRA...
42. Inactivation of surfactant may occur in this condition
43. A ground grass on chest radiograph is characteristic for this condition

Apnea of prematurity can be divided into


A. Central apnea
B. Obstructive apnea and
C. Mixed apnea which is a combination of these two events.

Match the definition in questions of the types of apneas in the questions 44-45
below to the classifications. Each option may be used once, more than once or
not at all.
44. Absence of noticeable airflow but with the continuation of chest wall
movements
45. Cessation of airflow and respiratory efforts with no chest wall
movement

46. Which of the following is the mainstay of pharmacological treatment


of apnea of prematurity?
A. Major tranquilizers
B. Opioids
C. Benzodiazepines
D. Methylxanthines
E. Neuroleptics

47. The incidence and severity of respiratory distress syndrome (RDS),


intraventricular hemorrhage, and necrotizing enterocolitis can be decreased by
the use of
A. Antenatal antibiotics
B. Antenatal corticosteroids
188 | P a g e ...TROY MADAH KAMPIRA...
C. Antenatal malarial prophylaxis
D. Antenatal VDRL screening
E. Antenatal Iron supplementation

48. Women at risk for imminent delivery of a fetus less than 32 weeks’
gestation need to receive neuroprotective therapy to reduce the risk of
moderate or severe cerebral palsy in the preterm baby. Which of these
medications will serve this purpose?
A. Antenatal corticosteroids
B. Magnesium sulphate
C. Prostaglandins
D. Calcium channel blockers
E. Beta blockers

Match the genetic syndrome A-B below with their characteristics or associations
in questions 49- 60.

A. Down syndrome
B. Turner syndrome

49. Coarctation of aorta


50. Brachycephaly
51. Shield chest
52. Maternal age is not a predisposing factor
53. Congenital lymphedema
54. Endocardial Cushing defects
55. Increased carrying angle of elbow (cubitus valgus)
56. Joint hyper flexibility
189 | P a g e ...TROY MADAH KAMPIRA...
57. Wide spread nipple
58. Protruding tongue, open mouth
59. Wide gap between 2nd and 3rd toes
60. Atlantoaxial instability

Match the types of bleeding disorders (A- D) with their manifestations in


questions 61-65 below. Each option can be used once, more than once or not at
all.
A. Hemophilia A
B. Hemophilia B
C. A and B
D. von Willebrand disease

61. It is primarily a deficiency of coagulation factor IX


62. It is primarily a deficiency of coagulation factor VIII
63. Results from deficiency or abnormality of a factor which initiates
platelet adhesion in primary clot formation.
64. Clinical manifestations are more common in males
65. Deep spontaneous bleeds are uncommon

Match the mechanism of disease (A-D) to the pathology found in patients with
sickle cell disease in questions 66 –69. Each option may be used once, more than
once or not at all.
A. Vaso-occlusion
B. Hemolysis
C. Parvovirus infection

190 | P a g e ...TROY MADAH KAMPIRA...


D. Auto splenectomy

66. Cholelithiasis
67. Hand-foot syndrome
68. Meningitis
69. Aplastic crisis

70. When do the classic clinical features of congenital hypothyroidism


usually appear?
A. Usually appear at birth
B. Usually starts to manifest in utero
C. Usually appear gradually over several weeks after birth
D. Usually appears in toddler years
E. Usually appears just before the child starts school

Suspected answers for the exam paper above


1. B 36. B
2. C 37. C
3. E 38. D
4. D 39. C
5. A 40. d
6. d 41. C
7. D 42. c
8. D 43. A
9. C 44. B

191 | P a g e ...TROY MADAH KAMPIRA...


10. B 45. A
11. A 46. D
12. B 47.b
13. B 48. B
14. B 49. B
15. B 50. A
16. D 51. B
17. B 52. B
18. B 53. B
19. B 54. A
20. A 55. B
21. A 56. A
22. B 57. B
23. A 58. A
24. D 59. A
25. C 60. A
26. B 61. B
27. A 62. A
28. B 63. D
29. A 64. C
30. C 65. D
31. A 66. B

192 | P a g e ...TROY MADAH KAMPIRA...


32. B 67. A
33. B 68. D
34. B 69. C
35. C 70. c

ANOTHER EXAM PAPER WITH PAEDIATRICS QTNS

Match the following types of abdominal pain A—B to their characteristics in


questions 1 -. Each option may be used once, more than once or not at all.
A. Visceral pain
B. Parietal pain

1. Usually dull and diffuse


2. Poorly localized
3. Can be elicited by cough
4. Usually sharp in intense

Match the following causes of referred sites of pain A- D to the most likely primary
abdominal cause in questions 5-8
A. Subscapular pain
B. Shoulder pain
C. Upper leg or groin pain
D. Back pain

5. Cholecystitis
6. Splenic injury
7. Ureteral colic (stones)
8. Pancreatitis
193 | P a g e ...TROY MADAH KAMPIRA...
Match the following causes of abdominal pain listed A-E to the likely presentation
in question 5- 13 below. Each option may be used once, more than once or not at
all.
A. Duodenal ulcer
B. Appendicitis
C. Intussusception
D. Urolithiasis
E. Pyelonephritis

9. A 12-year-old boy presenting with severe burning, gnawing pain. The pain is
relieved by food, milk. His uncle also has a similar problem.

10. A 10-year-old by presents with a one-day history of abdominal pain which


initially was periumbilical in nature but has localized to the right lower quadrant
in the 6 hours.

11. A 7-months-old by presents with a 1-day history of intermittent irritability. The


mother says she the child is passing frequent small volumes of bloody-mucoid
stool. On examination he seems to be a maintain a guarded position with knees
pulled up.

12. A 11-year-old girl presents with an acute onset of abdominal pains, frequent
urination and painful urination. On examination she is febrile with a
temperature of 39 °C and has costochondral tenderness.
13. A 10-months-old bay who was previously well presents with a 1-day history of
sudden onset of severe, diffuse pain. On examination he has a soft, nontender
mass in the right upper quadrant of the abdomen.
Match the following physical examination signs or test A – C to the associated
pathology in question 14- 16
194 | P a g e ...TROY MADAH KAMPIRA...
A. Murphy’s sign
B. Carnet test
C. psoas sign
14. Perinephric abscess
15. Acute cholecystitis
16. Abdominal wall pain

Match the following causes of abdominal pain A-B to their characteristic findings
in questions 1723.
A. Functional Constipation
B. Hirschsprung disease

17. Age of onset is usually < 1-year-old


18. There is usually a history of passage of meconium beyond 24 hours of age
19. Abdominal pain is frequent and colicky
20. It is accompanied by stool withholding behavior
21. It is usually accompanied by encopresis (soiling)
22. On rectal is usually empty on physical examination

23. In which of the following infectious causes of diarrhea are antibiotics


administration contraindicated?
A. Diarrhea caused by enterohemorrhagic Escherichia coli
B. Diarrhea caused by salmonella typhimurium C. Diarrhea caused by
salmonella enteritidis
D. Diarrhea caused by shigella sp.
E. Diarrhea caused by cholera sp.

195 | P a g e ...TROY MADAH KAMPIRA...


24. Which of the following medication is routinely recommended for treatment of
diarrhea and has been shown to reduce duration and severity of diarrhea in
children older than 6 months?
A. Ondansetron
B. Loperamide
C. Zinc
D. Aluminum trisilicate
E. Ciprofloxacin

25. Which antiemetic is safe for the treatment of vomiting in acute gastroenteritis?
A. Prochlorperazine
B. Ondansetron
C. Domperidone
D. Metoclopramide
E. Promethazine

26. What is the mechanism of action of the antiemetic drug in question 25?
A. 5-hydroxytryptamine antagonist
B. 5-hydroxytryptamine agonist
C. H2 blocker
D. H2 agonist
E. H3 blocker

27. A 4-year-old boy developed acute onset of severe abdominal cramps and
watery diarrhea, followed by grossly bloody stools and emesis. He was treated
with antibiotics and oral rehydration salts at a health centre. He presented
again a week after the onset of diarrhea with swelling of the face and feet with
little passage of urine. A full blood count showed low FBC and decreased
platelets. Which of the following is the most likely diagnosis?

A. Disseminated intravascular coagulation


196 | P a g e ...TROY MADAH KAMPIRA...
B. Severe bacteremia
C. Systemic inflammatory response syndrome
D. Hemolytic uremic syndrome
E. Acute tubular necrosis

28. What is the most likely organism that has caused the condition in question 27?

A. E. coli 0157: H7
B. Salmonella
C. Entamoeba histolytica
D. Campylobacter
E. Y. Pestis

29. You were seeing a 9-year-old boy presented with fever, abdominal pain,
nausea, decreased appetite together with your supervisor. The history indicated
that over the first week of these symptoms he also developed constipation. On
examination he had a fever 0f 38.5 degrees and his pulse was 75beat/min. Your
supervisor told you that his heart rate seems be paradoxical in relation to the
degree of the fever. He called this Faget sign or sphygmothermic dissociation.
You also did a full blood count which shows leukopenia. Upon further inquiry,
he said that he developed diarrhea approximately a week after the initial
symptoms began. Which of the following best describes this condition?

A. Shigellosis
B. Enteric fever
C. Hemolytic uremic syndrome
D. Campylobacter jejuni infection
E. Infection by Yersinia enterocolitica

Match the following types of bloody stool A-C to the location of bleeding in
questions 30-32 below
197 | P a g e ...TROY MADAH KAMPIRA...
A. Dark maroon stools
B. Black, tarry stools
C. Bright red blood spotting of stool noted on toilet tissue

30. Anal fissure


31. Peptic ulcer
32. Intussusception

33. The most common cause of hematemesis in a healthy term infant is swallowed
maternal blood. Which of the following tests can you do to differentiate
maternal from infant blood? A. Schilling test
B. Apt test
C. Clinitest
D. Microscopy
E. Osmotic fragility
34. An infant with known GER who periodically arches his or her back may have
what syndrome?
A. A syndrome associated with cerebral palsy
B. An epilepsy syndrome
C. Sandifer syndrome
D. Down syndrome
E. Turner syndrome

35. What is the most common type of intussusception?


A. Duodeno-jejunal intussusception
B. Jejuno-ileal intussusception

198 | P a g e ...TROY MADAH KAMPIRA...


C. Ileo-colic intussusception
D. Ceco-cecal intussusception
E. Colo-colic intussusceptions

36. Which of the following is the standard study to make this diagnosis
intussusception?
A. Supine abdominal x-ray
B. Erect abdominal x-ray
C. Abdominal CT scan
D. Abdominal ultrasonography
E. Barium enema

37. What is the classic radiographic finding in duodenal atresia?


A. Free air under the diaphragm
B. Fluid air-levels
C. Double-bubble sign
D. String sign
E. Pneumatosis intestinalis

Match the following types of rashes A- I against their characteristics in questions


38-49 below.
A. Scarlet fever
B. Scalded skin syndrome
C. Measles
D. Chicken pox
E. Rash associated with mononucleosis
F. German measles
G. Fifth disease

199 | P a g e ...TROY MADAH KAMPIRA...


H. Henoch–Schoenlein purpura
I. Erythema marginatum

38. Rheumatic fever is a complication of this condition


39. Rash may be elicited by administration of amoxicillin
40. The rash has a sand paper rash feel
41. May be treated with acyclovir
42. Positive Nikolsky sign is positive in this condition
43. Can cause fetal embryopathy
44. This is rash is a manifestation of acute rheumatic fever
45. Disseminated intravascular coagulation is a complication of this condition
46. Delayed subacute sclerosing panencephalitis is a complication of this condition
47. The World Health Organization recommends treatment with vitamin A in this
condition.
48. Rash distributed predominantly over the buttocks and lower extremities are
typical for this disorder
49. Can cause aplastic crisis in patients with chronic hemolytic anemia

Suspected answers for the paper above


1. A 25. B 49. G
2. A 26. A
3. B 27. D
4. B 28. A
5. A 29. D
6. B 30. C
7. C 31. B
8. D 32. A

200 | P a g e ...TROY MADAH KAMPIRA...


9. A 33. B
10. B 34. C
11. C 35. C
12. E 36.E
13. D 37. C
14. B 38. A
15. A 39.E
16. B 40.A
17. B 41. D
18. B 42. B
19. B 43. F
20. A 44. I
21. a 45.H
22. B 46. C
23. B 47.C
24. c 48.H

Clues to some of the MCQs that came during some of the Paediatric rotations
• Air under the diaphragm in salmonella typhi
• Early complication in DKA.: cerebral oedema/shock
• Cystitis staining-gram negative…cocci? bacilli?
• Treponema pallidum staining gram negative spirochaete

201 | P a g e ...TROY MADAH KAMPIRA...


• Failing to blow chick and closing eyes, which nerve? CN 7
• vocal cords always abducted…which nerve? CN 10
• Tonsillitis which vaccine? Diphtheria (DPT)
• Measles in a picture question
• Septic arthritis…. which bacteria…what stain is Steph. aureus? gram positive
cocci in clusters
• Malnutrition……low height for age and low weight for age……which type
of malnutrition? Severe malnutrition
• CSF culture. clear color, low glucose, raised proteins, India ink neg, gram
stain neg, high opening pressure…………what management? Ceftriaxone,
fluconazole, Avcrovir, TB treatment,
• Sickle cell presentation and history
• Acute diarrhea in children over 5 years old: Rotavirus
• Commonest cause diarrheal death worldwide is Rotavirus
• Comparison of bleeding in hemophilia and thrombocytopenia…. into
joints, nose bleeding, gums, intracranial
• Initial management of a trauma patient
• Maintenance fluids….32kg patient: 72ml/hr.
• developmental milestone of a 2 years old……draw circle, runs well, drinks
from a cup, follows 2-3 step commands,
• Hep B in pictures

202 | P a g e ...TROY MADAH KAMPIRA...


• 3 days old, bilious vomiting, abdominal distention poorly feeding but
passed meconium after birth,,,,,,,,, diagnosis duodenal atresia
• Pain in the umbilical area which radiated to the RLQ……diagnosis
appendicitis
• Increased ICP, diplopia, fever, neck stiffness,,,,,,,,,,,,,,, diagnosis meningitis
• Machinery murmur, bounding pulses……findings on echo
• 8 months with a hx of coryza and cough…………...PCP
• Bronchiolitis
• Pleural effusion and straw-colored fluid were drained…...diagnosis TB

PAEDIATRICS OSCE STATIONS

2020 OSCE STATIONS

1. Respiratory Exam

2. History taking of a 5-year-old male Child presenting with Cough and


fever for two days

3. Video of a convulsing child


i. Describe the movements that you see
ii. If you were unable to access the veins, what drug would you
give the child and through what route
iii. What one immediate management would you do for the child
iv. If the seizures were not able to be controlled for about 30
minutes, what drug would you give

203 | P a g e ...TROY MADAH KAMPIRA...


v. Mention investigations that you could do that can be done in
Malawi

2020 END OF YEAR DIFFERED PAEDIATRICS OSCE STATIONS

STATION 3: Cardiovascular exam

STATION 6: Abdominal exam

2019 END OF YEAR DIFFERED PAEDIATRICS OSCE STATIONS

STATION 2: A video having two clips


A. A new born baby inside a container
I. Write the examination respiratory notes of the baby (4 marks)
II. Mention 2 diagnosis of the baby (1 mark)

B. A two and half month child


I. Write the examination respiratory notes of this child (4 marks)
II. mention 2 complications of the respiratory problem (1 mark)

STATION 3: A 15 years old girl who was hit by a car and sustained an abrasion of
the head and had a deformed upper left upper limb. The airway was patent, had
no chest injuries and was breathing well. Other parameters were oxygen
saturation 96% RA, GCS 7/15, RR 30, PR 140bpm, BP 90/70.
a) Why does the girl have low BP and increased pulse rate? (2 marks)
b) Steps in the management of the girl? (6 marks)
c) What does the GCS mean? (3 marks)

204 | P a g e ...TROY MADAH KAMPIRA...


STATION 8: Can you do a neurological exam of the upper limb? (A child was
normal, just be familiar with what you have to look for and do)

STATION 12: You are asked to do a cardiovascular exam of a child. Make sure
you pick a murmur (why is it systolic or diastolic, where is it best heard, where is
it radiating to axilla, at the back and the carotids?)

2018 END OF YEAR PAEDIATRICS OSCE STATIONS

STATION 1: Cardiovascular exam


STATION 2: History taking- child presenting with breathlessness
STATION 3: NRU assessment

2016 END OF YEAR PAEDIATRICS OSCE STATIONS

205 | P a g e ...TROY MADAH KAMPIRA...


2014 END OF PAEDIATRICS ROTATIONS OSCE STATIONS
STATION 1: Video clip showing a malnourished child with severe dehydration 9
months old,7kg.

206 | P a g e ...TROY MADAH KAMPIRA...


a. State significant clinical signs seen in the child (features of severe
dehydration)
b. If the child is malnourished and shocked what is the fluid of choice
c. How much of the fluid do you give in the first 30min and via what
route?
d. If the child is NOT malnourished and NOT shocked, what fluid do
you give and via what route? How much in first 30 min?

STATION 2: Neonatal resuscitation

You are the intern on call, a baby is born but can’t breathe despite efforts to
resuscitate, what are you going to do and do it now?

STATION 3: Communication
a. A child is kwashiorkor and shocked. and child can not drink and nurses
have tried to cannulate but have failed. He certainly needs NGT but
mother is refusing doctors to put an NGT because she believes her
previous child died with NGT. Can you please convince the mother to
accept the NGT

STATION 4: History taking

• Mrs. Phiri has a sick child. Can you please have a chat with her and
take a detailed history of the child tell us the story and your
differentials at the end?
• Cough
• Seizure
• abdominal

STATION 5: Clinical examinations- any system (15 min)


207 | P a g e ...TROY MADAH KAMPIRA...
• Examine, differentials, Ix and Mx and answer questions

2013 END OF YEAR PAEDIATRICS OSCE STATIONS

1) Trauma case (Dummy - ABC and ETAT)


2) Neuro examination (lower limb)
3) Convulsing child History taking
4) Clip of a child with signs of severe SOB (know + DDx)

2011 PAEDIATRICS OSCE STATIONS IN VIDEO FORMAT

Instructions:

Watch the video clips (1 and 2) and answer the questions below.

The clips are on a continuous loop lasting about 1 ½ minutes each.

Clip 1

This is a preterm infant born at 28 weeks gestation.

a. Write an examination note regarding his respiratory system (4 marks)

i. RR 66 (range 60 to 70)
ii. “See saw breathing”
iii. Subcostal recession
iv. Sternal recession
v. Intercostals recession
vi. Receiving Oxygen via a nasal catheter

b. Suggest 2 common causes of this appearance in a preterm infant. (2 marks)

i. Surfactant deficiency/ respiratory distress syndrome/ Hyaline Membrane disease

208 | P a g e ...TROY MADAH KAMPIRA...


ii. Pneumonia
iii. Pneumothorax

c. Above what respiratory rate would you consider a child to have fast breathing
according to the WHO classification, for each of the following age ranges?
a. Child less than 2 months of age (1 mark)
RR≥ 60 breaths per minute

b. Child between 2 and 11 months of age (1 mark)


RR≥ 50 breaths per minute

c. Child between 1 year and 5 years of age (1 mark)


RR≥ 40 breaths per minute

Clip 2

This is a two-month-old child who has been admitted with pneumonia

a. Write an examination note regarding his respiratory system (4 marks)


i. RR=80 (range 70 to 90)
ii. Subcostal recession
iii. Tracheal tug
iv. Intercostal recession
v. Receiving Oxygen via nasal prongs
vi. “See saw breathing”
vii. Lethargy

b. List four features of pneumonia that would lead it to being classified as very
severe according to the WHO classification. (4 marks)
i. Central cyanosis
209 | P a g e ...TROY MADAH KAMPIRA...
ii. Inability to breast feed or drink, or vomiting everything
iii. Convulsions, lethargy or unconsciousness
iv. Severe respiratory distress

c. List three complications or other diagnoses you would consider in a two-


month-old child who has been treated appropriately for pneumonia but has not
improved after two days (3 marks)
i. Empyema/pleural effusion
ii. TB
iii. PCP
iv. Resistant organism
v. Staphylococcal pneumonia
vi. Foreign body
vii. Aspiration pneumonia
viii. Cardiac failure
Other Stations
3. Explain how to use a metered dose inhaler (MDI) with a spacer device
i. Cut a small hole in the bottom of the plastic bottle
ii. Fit the inhaler mouthpiece with an airtight seal
iii. Shake the inhaler before each puff
iv. Shake the canister
v. Exhale functional residual capacity
vi. Place the spacer device nozzle to open mouth
vii. Inhale slowly but deeply
viii. Close the mouth then hold breathe for 10 seconds then exhale
ix. Repeat the process.

4. System examination:
i. Cardiovascular
ii. Respiratory
iii. GIT
210 | P a g e ...TROY MADAH KAMPIRA...
iv. Neurological (Lower limb)

5. History taking:
i. Convulsing child
ii. Abdominal pain

211 | P a g e ...TROY MADAH KAMPIRA...

You might also like